Act English Skills
Act English Skills
Act English Skills
One of the trickier concepts that you will be tested on ACT English is your ability to
choose the right word from among many that are not quite right, and to spot when
words are being used incorrectly.
Are you confident in your ability to find the differences in a group of similar words? Can
you distinguish what’s needed from the context of a question?
If you’re not so sure, read on to see how the ACT English will test you on this skill! In
this post, I'll cover everything you need to know about these questions:
On the ACT English, there are three main ways that word choice becomes important:
This final topic is large enough that we have a whole separate article dedicated to it.
But for the first two, let's look at some example sentences that illustrate these concepts.
1. A. NO CHANGE
C. better
D. better to
2. A. NO CHANGE
B. simulating
C. dazzling
D. stimulating
Answers: 1. B; 2. D
How did you do? These questions cover two of the most common types of diction errors
that you will see on ACT English – commonly confused words and understanding
meaning in context. Both of these test your ability to understand when a word is being
used incorrectly. I’ll go into each of these question types in more detail below.
But first, let’s talk about why ACT English has diction errors in the first place.
Most of the ACT English questions focus on grammar, punctuation and style. By
reading our guides or using another prep method, you can learn the various grammar
rules that are important for being able to answer most of the questions on ACT English.
With diction questions, the ACT is looking to see which students can also spot
errors where you can’t apply your grammar rules. The only way you will know these
answers is to understand the subtle differences between similar words.
If you don’t think this task sounds too difficult, keep in mind that many students make
diction errors all the time in their everyday speech and writing. The makers of the
ACT know which words are most confusing for students, and focus on these when
writing the test.
So let’s start there – with the most commonly confused words that the ACT loves to test.
However, the ACT writers have a couple of favorite errors, which usually appear at least
once on every test. We'll go through these one at a time.
The first key concept for diction questions is understanding the difference between
“than” and “then.”
I am smarter than you are.
Pop Quiz!
Look at the following pair of sentences. Can you tell which one is correct?
Answer: the second is correct because the sentence shows a comparison - than is
needed.
A. No change
B. better than
C. better
D. better to
Now you can understand why the answer is (B) better than. The sentence shows a
comparison (waking up and smelling coffee vs. waking up and doing anything else), so
we need than.
HAVE vs. OF
The second most important diction pair to know is “have” and “of."
The important distinction here is that “have” is a helping verb, while “of” is a preposition.
If you see “of” being used as a helping verb, it will be incorrect!
If you see could of, would/will of, should of, or might of: these are all INCORRECT.
This is one of those “everyday English” mistakes. In speaking, people tend to
pronounce “have” like “of” because it’s faster and easier to say. It also sounds a lot like
our contractions “would’ve," “should’ve," etc.
This has led some people to believe that “of” is actually the correct word to use in these
circumstances - but it’s not!
She would of preferred gorgonzola, but she got Cornish blue instead. INCORRECT
She would have preferred gorgonzola, but she got Cornish blue instead. CORRECT
Have/of questions should also be very easy to spot if you're looking for them. If you
see of or have underlined, be ready to see a diction question.
Remember that if you have would/will of, might of, should of, or could of, you
can immediately cross out those answers.
Choose an answer that replaces “of” with “have” and is also grammatically correct in the
sentence.
Dickinson’s last twenty years of letters -- many over 1,500 words in length -- reveals the
breadth and depth of her connection to the world through a wide circle of
correspondents.
A. NO CHANGE
B. reveal
C. will of revealed
D. would of revealed
First thing we see in the answer choices are two answers that can automatically be
crossed out, which makes our lives so much easier! Get rid of C and D, which both
have the dreaded “of” construction.
Now it’s a matter of differentiating between “reveals” and “reveal." At this point, you may
notice that we are looking at a subject-verb agreement question. So we need to find
what the subject is.
Ask yourself: what is doing the revealing? It’s the “last twenty years of letters." When we
cross out modifiers and the prepositional phrase, we get “last twenty years of
letters." Now it’s obvious that “years” is the subject.
Since “years” is a plural noun, we need the plural verb, which is “reveal." B is the
answer.
The ACT will rarely test other commonly-confused words. These questions are very
difficult to predict because they happen quite infrequently.
I have included a list of commonly confused words at the end of this article. Try reading
through them and see if there are any that surprise you or that you didn’t know.
If so, it wouldn’t hurt to make sure you have these down before you take the test.
But don’t bother spending a lot of time worrying about this list. The ACT will rarely test
this kind of question, so it’s really not worth stressing out over.
You will most often see these commonly confused words in the context of our next type
of question, which is….
This can be tricky because you have to really think about what the differences are
between the different words that you're given, and also understand what the sentence
needs in order to most correctly complete it.
As a child growing up in two cultures, I found this contest a bit confusing. When I was in
the fifth grade, was I ten or eleven years old?
A. NO CHANGE
B. change
C. dispute
D. difference
This example shows how the ACT uses relatively simple words to try to trick you. You
are probably familiar with all of the words in the answer choices, so let’s look at them
closely to see which one best fits the sentence.
All of the words here imply some sort of contrast or conflict, but in very different
ways.
Let’s start with the given word, “contest." A contest implies some form of formal
competition between two things. Though the author says that there are differences
between American and Korean age counting traditions, he doesn’t imply that they were
actually in competition with each other. So (A) is out.
Let’s look at (B). “Change” implies that something was one way, and then became
another way. This doesn’t work because these traditions haven’t changed from one to
the other - they were just different to begin with. So (B) is out.
(C) is quite similar to (A), in that “dispute," like “contest," shows a direct conflict between
two things. Again, this doesn’t really work because the two traditions aren’t actually
competing with each other.
That leaves us with (D), which is the only answer that makes sense. As we’ve said, the
two traditions aren’t having a fight; they contrast simply because they have different
characteristics. That means that (D), “differences," is the most appropriate answer.
The meaning in context questions are definitely the trickiest of the diction questions. To
spot them, look for questions that have an underlined word and answer choices
that are completely different words, but are loosely related in meaning.
First, read through the sentence. Try to place a word of your own in the spot of the
underlined word.
Now look at the answer choices. Though they will be slightly similar words, they will
have different meanings. Which of them can have the meaning most like the word
you put into the sentence? Choose that as the answer.
Let’s look at this example from the ACT and use the above strategy to answer it.
A. NO CHANGE
B. raised
C. lifted
D. lighted
First, let’s read through the sentence and try to see what should go in the blank.
“Perhaps the celebration of New Year’s Day in Korean culture is _______ because it is
thought of as everyone’s birthday party.”
Something along the lines of “made more important” makes sense here. So which word
could mean that?
You’ll notice that almost all the words imply going up in one way or another - this is how
the ACT is trying to trick you. But in the right context, one of them can also mean
showing greater importance.
“Raised” and “lifted” both refer to physically moving something up (unless they are
paired with other words - not the case here). So (B) and (C) are out.
“Lighted” is thrown in here to trick you and is quite different to the others - its means
something has been made lighter, as in the opposite of darker. It doesn’t work at all, so
(D) is out.
Therefore the best word is the one the sentence started out with. If you look up
“heighten”, you will see it can mean to physically raise something up, but it can also
mean to make something more intense or significant, which is close to the original idea
we came up with, which was making something more important.
Looking Back
The second question I gave you about smelling coffee in the morning had one of these
meaning in context errors. Take a second look at the question.
Though the answer choices are all vaguely related – they all imply something that
excites or surprises in some way – the correct answer is “stimulating”. This is the only
word that really implies something that helps wake you up, which is what is needed to
correctly complete the sentence.
Quick Recap
To summarize, here are the key strategies you need to use to master diction questions
on ACT English:
He slices a small piece off the outer edge of the wheel. A fine dusting of white mold
already covers the cheese. The flavor is intense and creamy, with just a hint of the tang
that will of developed (2) over the next five months with only Khoubbane's watchful eye
and sensible (3) palate to determine when it has finished maturing.
It is an unexpected undertaking for the 56 year old. Cheese, as the western world
knows it, is unpopular in Morocco. "Moroccans don't eat smelly things," he says with a
shrug. "There is less love for cheese here then in (4) France or Italy."
His success, he says, has been due to the fact that he has viewed the process as a
labor of love, rather than a business adventure.(5)
C. differentiated
D. specified
2. A. NO CHANGE
B. would of developed
C. will develop
D. develops
3. A. NO CHANGE
B. sensitive
C. emotional
D. touchy
4. A. NO CHANGE
B. then between
C. than in
D. than between
5. A. NO CHANGE
B. venture
C. advantage
D. proce
1. A; 2. C; B.; 4. C; 5. B
a certain production of
addition something added edition
something
altogether completely; entirely all together all things with each other
amoral having no sense or right and wrong immoral having intentionally bad morals
assent agreement/approval
breath air that comes from lunch breathe to take air in and out
to provide supporting
collaborate to work together corroborate
information
complacent feeling you don’t need to try hard complaisant willingness to please others
dominant most important or wanting control dominate to control or have power over
later occuring further along in time latter near the end of a period
pique annoyance
peddle to sell
perpetrate do something bad perpetuate to sustain or preserve something
thorough completely
waist around the middle of your body waste a bad use of something
where to, at, or in what place were past tense of "to be"
Transition Questions on ACT English: Strategies
and Practice
Transitions are one of the most common and trickiest rhetorical topics tested on the
ACT English section. However, there are a couple simple rules that can make
answering these questions much easier.
In this post, I'll cover everything you need to know to approach all three types of
transition question:
3 Types of Transitions
The ACT covers three basic transitional relationships: addition, contrast, and causation.
Understanding how these relationships work will be very helpful for the test.
Contrast: Words like "however" and "still" that introduce a conflicting point or idea.
Keep in mind that not every transition falls into one of these categories. They're more
what you'd call guidelines, than actual rules.
Nonetheless, thinking about transitions in these terms can help you pick out the right
answer on ACT English questions, as you'll see below.
Malcom and Sam were best friends. Even so, they spent every moment of the day
together.
Something about this transition doesn't quite make sense. "Even so" is a contrast
transition, but these to sentences aren't opposed to each other: if Malcom and Sam are
best friends, it makes perfect sense that they would spend every day together. Instead it
would make more sense to use a causation transition, or even drop the transition word
completely:
Malcom and Sam were best friends. As such, they spent every moment of the day
together.
Malcom and Sam were best friends. They spent every moment of the day together.
We're going to cover a step-by-step approach, key transition words, and some helpful
ACT English tips for questions about connecting sentences.
Step-by-Step Approach
#1: Cross out the underlined word. Always start by crossing out the word that’s there.
Otherwise, if it's not obviously wrong, you may be biased in favor of the original
phrasing.
#2: Read to the end of the sentence. You should be doing this on every question, but
it's especially important here to make sure you understand how the two sentences are
related to each other.
#4: What type of relationship is it? Addition, contrast, or causation? If you're not sure,
it can be helpful to think about whether you would connect the sentences with and
(addition), but (contrast), or so/because (contrast).
#5: Narrow down your choices. Once you have a sense of what you're looking for,
rule out any answers that don't make sense or that aren't grammatical.
#6: Plug answer into sentence to check. When you think you have the answer, plug it
into the sentence and make sure the transition is logical.
We'll walk through this process with an example shortly, but first let's go over some key
words and strategies that you need to know.
Key Words
Transitions between sentences are generally conjunctive adverbs, like "however" and
"furthermore," or prepositional phrases, like "for example" and "on the other hand." You
can see the most common ones below, sorted by type.
Introducing Sentences
You may also see options that are grammatically incorrect. These will generally be
coordinating or subordinating conjunctions (see the chart below for examples), which
can't be used immediately before a comma to introduce a complete sentence.
Helpful Tips
Beyond the basic concepts we've discussed, there are some ACT English-specific tips
that can really help you approach transition questions.
If two choices are synonyms, neither is correct. If two of the words mean the same
thing (they must be synonyms, not just belong to the same category), there's no way to
choose between them, so neither can be correct. When you see two choices that are
synonyms, rule them both out.
If one of the choices omits the transition word altogether, that's usually the
correct answer. Always check any answer that leaves the transition word out first—if
the paragraph works without it, that's the right answer.
Don't panic if the three categories don't apply. Not every question deals with
transitions that fit into the categories outlined above. Simply use the rest of the strategy
(thinking about how the two sentences are related, narrowing down choices that don't
make sense, and then plugging in the answer you think is right to check) to pick out the
best answer.
Watch out for questions that ask for the LEAST acceptable option. Make sure to
use process of elimination to rule out any answer that does work.
We've covered a lot of material on transitions—let's put it into practice on a real ACT
question.
This example is a case where the transition has been shifted into the sentence, but we'll
approach it in the same way. The first step is to look at the two sentences without the
transition:
Snowflakes form from tiny water droplets, following a specific process of chemical
bonding as they freeze, which results in a six-sided figure. The rare "triangular"
snowflake confounded scientists for years because it apparently defied the basic
laws of chemistry.
Does an obvious transition jump out at you? Not really. On to the next question!
How are these sentences related to each other? The first sentence describes how
snowflakes are formed. The second brings up a seeming exception to that rule. This
relationship is contrasting.
Rule out answers that don't work. "Additionally" and "similarly" are too similar to
choose between, so both must be wrong. "For example" doesn't make sense, since the
second sentence is actually about an exception to the rule laid out in the first, not an
example of it.
Plug in the remaining choice. We have only "however," which is a contrast transition,
left. Let's try it out in context:
Snowflakes form from tiny water droplets, following a specific process of chemical
bonding as they freeze, which results in a six-sided figure. The rare "triangular"
snowflake, however, confounded scientists for years because it apparently defied the
basic laws of chemistry.
That transition makes sense, so J must be the correct answer.
Connecting Clauses
Issues with transitions between clauses may also be tested in conjunction with correctly
connecting independent clauses, so watch out for punctuation as well. (For more details
on connecting independent clauses, check out our post on run-ons and fragments.)
Let's use the strategy above to answer a real ACT example that asks about transitions
between clauses:
First we have to look at the underlined word and answer choices and recognize that
these subordinating conjunctions, so this a question about connecting clauses and not
sentences.
Next, let's cut out the current transition word and break the sentence into its two
component clauses (I'm also going to eliminate the descriptor at the end for simplicity):
There's not much chance that a seven-year-old just learning the game can hit a pitched
baseball
Does an obvious transition jump out at you? I would use "so" or "because" to
connect these two ideas.
How are these sentences related to each other? Those transitions jump out at me
because the idea in the first clause (that a seven-year-old can't hit a pitched baseball)
clearly leads to the solution in the second (putting the ball on a tee). This connection is
a causal relationship.
Rule out answers that don't work. Neither "while" nor "although" can work because
they're interchangeable (and both contrast transitions). "Unless" doesn't make sense.
Given that all of the choices are true, which one would most effectively introduce the
main idea of this paragraph?
Which of the following sentences offers the best introduction to this paragraph?
The exact phrasing may vary, but these types of questions always ask about "transition"
or "introduction."
Step-by-Step Approach
Because it's impossible to predict the content of an entire paragraph, it's vital that you
use process of elimination for questions that ask about transitions between paragraphs.
Here's a step-by-step approach to help you narrow down your choices.
#1:Pay attention to what the question is asking for. Though many of these questions
simply ask for the sentence that provides the best transition or introduction, some may
give a more specific. Make sure to read the question carefully and think about what it's
asking.
#2: Read into the paragraph at least a couple of sentences. Ideally, you'll read the
whole paragraph before answering transition questions, but you absolutely must read at
least one sentence after the first to get a sense of the context.
#4: Look for anything that needs to be introduced because it's referred back to
later in the paragraph. In the subsequent sentences, look out for pronouns like this
and these that reference ideas or nouns that need to be introduced in the first sentence.
This will often be the best hint for the correct answer.
#5: Narrow down the choices. Rule out answers that don’t make sense or don’t fit with
the general tone of the passage.
#6: Plug in the sentence you think works best. When you've eliminated three
choices, read the last answer in context and check that it makes sense.
Not all of these steps will apply to every question—the important thing is to think about
these ideas as you work to rule out choices.
Let's walk through how to approach a paragraph transition question from a real ACT.
What is the question asking for? The best transition between the paragraphs.
Narrow down choices. We can immediately rule out F, because it's about the town and
not the pottery, which is the topic of these two paragraphs. G and J are both relevant to
the topic of the ancient pottery, but neither makes sense as a transition. The patterns
are initially described earlier in the passage, and Quezada's painting doesn't come up
until later in the second paragraph.
Plug in the remaining option. Let's look at the passage with H, the only remaining
option, plugged in.
Fascinated by geometric designs, Quezada wondered if he could make pots like these.
Quezada began working with clay from the mountains. He dug the clay, soaked it, and
tried to shape it into a pot.
Because you will most likely see parallel structure questions on the ACT, let’s learn this
rule and raise your ACT English score.
In this post, I'll do the following:
The basic parallel structure rule is that the things in a list should be in the same
grammatical form. If you're listing three things, the construction of that list should
be noun, noun, noun, or verb, verb, verb, or gerund, gerund,
gerund, etc. Any inconsistency within the list is an error in parallel structure.
The sentence is listing three things that LeBron James does with the basketball. The
first two things are verbs. The last thing, "blocking," is in the gerund form. A gerund is a
verb that acts like a noun and ends in "ing." All three things should be in the same
grammatical form.
To correct the error in parallel structure, the last item in the list should be in the verb
form, too. Here's what the sentence looks life after it's corrected:
The example from above was a parallel structure list sentence. Generally, in list
questions, three things are listed and you have to verify that all the items in the list are
in the same grammatical form. This is another "list" sentence that contains a parallel
structure error:
Martin Luther King is admired for his courage, his dedication, and being intelligent.
Can you recognize the parallel structure error? The first two items are nouns, traits that
people admire in MLK. In the last item, "being intelligent," the word "being" is
a gerund and "intelligent" is an adjective that describes MLK. We want all three items
in the list to have the same construction. After fixing the parallel structure error, this
is our sentence:
Martin Luther King is admired for his courage, his dedication, and his intelligence.
All of the list items are in the same form. The sentence reads better and the structure is
parallel. Let's go over some tips for how to figure out parallel structure list questions on
the ACT.
To correctly answer parallel structure list questions, first identify that there is a list of
items. Usually the list will look like this: x, y, and z. Make sure that the commas are
separating items in a list and are not just separating clauses. Once you have identified
the list, determine whether the items in the list are as consistent with the other items in
the list as possible. How do we do that?
Break down each item in the list by identifying the parts of speech of the words and
make sure that all the items match. We want each item to be in the same grammatical
form. After breaking down the list items, there should be no inconsistencies. Once we
fixed our previous example sentence, the items in the list included "courage" (noun),
"dedication" (noun), and "intelligence" (noun).
While the list questions are relatively basic, there's a second type of parallel structure
question that can be a bit more challenging.
Parallel structure phrase questions are slightly more complicated than list questions, but
they follow the same principle. The parallel structure rule regarding phrases is that the
construction of a phrase on one side of a conjunction must match the
construction of the phrase on the other side of the conjunction as closely as
possible.
Definition of a Conjunction
Additionally, there are correlative conjunctions, also known as word pairs. Literally,
these words come in pairs. The items correlative conjunctions compare follow each half
of the word pair. Examples of word pairs include either...or, not only...but also, as...as,
and both...and.
Example Sentences
Now that we know the rule and the definition of a conjunction, we can learn how this
rule should be applied. This is a sentence with a parallel structure error:
There are two items in the sentence: the two things that the ACT English section does.
The two items are connected by the conjunction "and."
Let's break down each item by the parts of speech of the words in each item. The first
item, "challenges students," is VERB + NOUN . The second item that follows the
conjunction is "frustration is found in them." That phrase's construction
is NOUN + VERB + ADJECTIVE + PREPOSITION + PRONOUN. Even if you struggle
identifying the parts of speech of certain words, you should be able to immediately
recognize that the phrases are not consistent and the sentence has a parallel structure
error.
So how do we correct the sentence? We change the wording in the phrase following
"and" to match the VERB + NOUN construction of the first phrase. This is the corrected
version of the sentence:
The two items in the sentence are two things that Kanye West displays. Those things
are connected by the word pair "both..and." Again, let's break down the parts of speech
of the words in those things. Phrase #1 is "reprehensible behavior at awards shows,"
and "reprehensible"=ADJECTIVE, "behavior"=NOUN, and "at awards
shows"=PREPOSITIONAL PHRASE. The basic construction of phrase #1
is ADJECTIVE + NOUN + PREPOSITIONAL PHRASE.
Phrase #2 is "on late night talk shows behaving bizarrely." What is the construction of
phrase #2? Well, "on late night talk shows"=PREPOSITIONAL PHRASE,
"behaving"=GERUND, and "bizarrely"=ADVERB. The construction of phrase #2
is PREPOSITIONAL PHRASE + GERUND + ADVERB. We want the constructions of
the phrases to match as closely as possible. Think about how you could change the
sentence to fix the parallel structure error. Here's the corrected version of the sentence:
I changed phrase #2 to match the construction of phrase #1. Both phrases are now in
the form of ADJECTIVE + NOUN + PREPOSITIONAL PHRASE. Check it out:
"bizarre"=ADJECTIVE, "behavior"=NOUN, and "on late night talk
shows"=PREPOSITIONAL PHRASE. Success! The constructions of the phrases match
and the sentence is easier to understand.
ACT English Strategy
If you see a conjunction connecting or comparing two items, identify the items. Then,
break down the words in each item by their parts of speech and determine each
phrase's construction.
Make sure that the phrases are parallel. The construction of the phrases should match
as closely as possible. Also, a preposition used on one side of a conjunction or
word pair must appear on the other side. Look at this example with a parallel
structure error:
The mixed martial artist was complimented not only for his tenacity but also in his
technique.
In the above sentences, the prepositions "for" and "in" correspond with "complimented."
The sentence is saying that the mixed martial artist was "complimented for" this and
"complimented in" that. Hopefully, your knowledge of idioms would tell you that it's
incorrect to use the expression "complimented in." Even if you weren't aware of the
idiom error, to keep the sentence consistent and follow parallel structure rules, use the
same preposition before and after "but also." This is how the sentence should look:
The mixed martial artist was complimented not only for his tenacity but also for his
technique.
Let's apply these strategies and our knowledge of parallel structure to actual examples
from the ACT English section.
As a young woman, she wrote of pining for a valentine and of visiting the Chinese
museum in Boston.
F. NO CHANGE
G. visiting to
H. of her visiting to
J. of her visiting at
Explanation: There is nothing grammatically incorrect within the phrase "of visiting."
Let's look at the phrase's function within the sentence. It follows the conjunction "and,"
and it's one of two things that she wrote about. For the sentence to be parallel in
structure, the phrases before and after the conjunction should have as close to the
same construction as possible.
Phrase #1 is "of pining." After breaking down the words in the phrase, we can determine
that the construction of that phrase is "OF" + GERUND. Phrase #2, "of visiting," also
has the construction "OF" + GERUND. The phrases are parallel. The answer is F. If
you're wondering why I didn't include "for a valentine" and "the Chinese museum in
Boston," those phrases just provide descriptive information about what she pined for
and where she visited; they do not alter the basic construction of the sentence.
Others, salt-encrusted, "sleep" in ancient caverns, waking after centuries to feed and to
be bred.
A. NO CHANGE
B. for breeding
C. to breed
D. breeding
Explanation: There's nothing wrong within the phrase "to be bred," but what's the
phrase's function within the sentence? It's one of two listed reasons why the others
wake after centuries. The reasons are connected by the conjunction "and." Reason #1
is "to feed." The construction of that phrase is simply an INFINITIVE ("to" + verb).
Reason #2's construction is INFINITIVE + PAST PARTICIPLE. Even if you were unsure
of the part of speech of "bred," you should notice that the construction of reason #2 is
inconsistent with that of reason #1. To make the phrases parallel, reason #2 should be
in the infinitive form. The answer is C.
Here are some general tips to help you correctly answer parallel structure questions on
the ACT.
When you see a list of items, make sure that all those items are in the same
grammatical form.
#2: Make Sure Phrases that Appear Before and After Conjunctions are
Parallel
Identify the parts of speech of words within a list or phrases that precede and succeed a
conjunction. Make sure the construction of the items matches as closely as possible.
I've created some realistic practice problems to test you on your knowledge of parallel
structure.
Additional Practice
Apply what you've learned in this article to help answer the following realistic ACT
parallel structure questions.
A. NO CHANGE
B. as in Ethiopia
C. than Ethiopia
D. as inhabit Ethiopia
2. Unsurprisingly, the diligent student completed his homework punctually, studied the
material thoroughly, and his presentations were delivered well.
A. NO CHANGE
B. his presentations were well delivered
C. delivered his presentations well.
D. his delivery was good on his presentations.
3. Greg reached his dream not only of having a family but also of becoming a
successful entrepreneur.
A. NO CHANGE
B. to become
C. becoming
D. became
4. Before leaving your house on the morning of the ACT, make sure you have pencils, a
snack, and remembering a calculator.
A. NO CHANGE
B. a calculator
C. to remember a calculator
D. have had remembered a calculator
5. Because Matt does not get enough sleep, he has difficulty focusing and to stay alert.
A. NO CHANGE
B. stays
C. staying
D. OMIT the underlined portion
Answers: 1. D, 2. C, 3. A, 4. B, 5. C
Verb Tenses and Forms on ACT English: Grammar
Rule Strategy
Define the verb tenses and forms that are tested on the ACT.
Provide information about when to use different verb tenses.
Detail how to construct verbs in different tenses.
Explain and demonstrate how verbs are tested on the ACT.
Provide practice questions to test you on what you’ve learned.
Present
Function
The present tense is the verb tense you use when you're talking about things that
are currently happening or things that are considered facts. Examples of verbs in
the present tense are “jumps," “sings," and “explain.”
Also, the present progressive is considered a form of the present tense. The present
progressive is formed with the present tense of “to be” + the gerund (“ing”) form of the
word. Examples include "am explaining," "is running," and "are laughing." Typically,
words like “currently” or “now” indicate that you should use the present tense of a verb.
Construction
Singular Plural
I jump We jump
You jump You jump
He/She/It jumps They jump
The present tense is not specifically tested on ACT English, but you do need to know
how to properly conjugate verbs in the present tense for subject-verb
agreement questions.
Simple Past
Function
Construction
Typically the simple past tense of a verb is formed by adding “ed” to the verb. The past
tense of “play” is “played.” The past tense of “listen” is “listened” and the past tense of
“discuss” is “discussed.” Here's an example sentence with a verb in the simple past
tense:
Many verbs don't follow this construction and the past tense is formed irregularly. For
example, "buy" becomes "bought," "come" becomes "came," and "grow" becomes
"grew."
Present Perfect
Function
Use the present perfect tense for actions that began in the past but are still
continuing in the present.
Construction
The present perfect is formed with has/have + the past participle. For regular verbs,
the past participle is formed by adding "ed" to the verb. Examples of present perfect
verbs include "has talked," "have done," and "has brought."
For the past seven hours, George has listened to Miley Cyrus songs.
As shown in this example, the words “for” and “since” often indicate that the present
perfect tense is needed. Be aware, however, that there are other instances when
context determines that you should use the present perfect tense.
Past Perfect
Function
When a sentence describes two completed actions, the past perfect is used for the
action that came first.
Construction
The past perfect tense is formed with had + the past participle. Examples of past
perfect verbs include "had talked," "had danced," and "had grown."
Take a look at this sentence that shows the past perfect tense used correctly:
By the time his mom came home from work, George had listened to Miley Cyrus
songs for seven hours.
Now that we're familiar with these basic verb tenses, we have the necessary foundation
to discuss exactly how verb tenses and forms are tested on the ACT.
Consistency
Most of the verb tense questions on the ACT English section have to do with
consistency. The basic consistency rule regarding verbs is that verbs should remain
consistent in tense or form throughout a sentence. Sentences that start in the past
should stay in the past and sentences that start in the present should stay in the
present. Here's an incorrect sentence that doesn't follow the consistency rule:
Maria studies science and played lacrosse.
The verb “studies” is in the present tense and “played” is in the past. The verb tenses
should be consistent. This is the corrected version of the same sentence:
Maria studies science and plays lacrosse.
We could have corrected the sentence by changing "studies" to "studied". The important
thing to remember is that the verb tenses should be consistent.
Sometimes, however, you can have a shift in tense and the sentence can still be
correct. This kind of construction is only possible if the verbs are in different clauses.
Take a look at these examples:
Justin bought a Honda and saves money on gas.
The first sentence is incorrect since "bought" and "saves" are in different tenses but the
same clause.
The second sentence, on the other hand, is correct. The tense shift takes place in a
different clause and the two verbs are occurring at different times: Justin bought the car
in the past, but he can save money on gas in the present.
On the ACT, the verb tenses of surrounding sentences can provide context clues for the
proper tense to use in a given sentence to maintain consistency. Take a look at this
example:
The shift from the present tense, "likes," in the first sentence to the past tense, "was," in
the second sentence doesn't make sense in context. The tenses should remain
consistent. Here's the correct version of the sentences.
Now, let's go over some tips for answering ACT questions that test verb tense
consistency.
Strategy
Here's some strategic advice for you: if a verb is underlined and the answer choices are
different tenses of the same verb, look at the surrounding sentences (a sentence or
two before and after) for context clues about the proper tense to use. If there are
multiple verbs in a sentence, identify the tenses to make sure they're consistent. If
there's a shift from past to present or vice versa, determine if the variation is acceptable
given the context of the sentence.
Use these tips to answer the following example from a real ACT.
Rely on your verb knowledge to figure out this actual ACT English question.
Explanation: From the answer choices, we can tell that we're most likely dealing with a
verb tense question because three of the four answer choices are in different verb
tenses: "they were," "they would," and "they're."
Whenever you have an underlined verb, check for possible verb tense errors. The verbs
"continue" in the first sentence and "score" in the second are in the present tense. The
shift to the past tense, "were," doesn't make sense given the context.
Here's another example of an actual ACT question that tests verb tense consistency.
Go through the same process that we went through in the previous question to answer
this ACT English question.
Explanation: This is an obvious verb tense question. Only the verb is underlined and
each answer choice is in a different tense.
In the first sentence, the simple past tense verb “encountered” indicates that we’re
referring to completed actions. However, the present perfect verb “have borrowed” can
only be used for an action that is still happening. The context of the sentence implies
that the sentence is referring to a completed action and the verb tenses in the two
sentences should be consistent.
Once you identify that this is a consistency question, you can immediately eliminate any
answer choice that isn't in the simple past tense. After eliminating answer choices, we're
left with J.
Here's one final verb consistency example for you. Because tense consistency
questions are the most common verb form questions, I want to make sure you fully
understand them.
Employ your verb expertise to figure out the correct answer to this real ACT question.
Explanation: Once again, we're dealing with an obvious verb tense question. Only the
verb "have" is underlined and the answer choices are all different verb tenses. The
verbs “took” in “took part” and "conducted" are in the simple past tense. The first
sentence is referring to completed actions that took place from 1942-1945.
Based on context, we can imply that the verb “have” is referring to what happened
before 1942. Therefore, we should use the past perfect tense because we are referring
to the completed action that came first. Instead of “have been using,” the correct verb
form is “had been using.” The answer is G. Even if you didn't recognize that you
needed to use the past perfect tense, you could have recognized that you needed
to change "have" to the past tense due to verb consistency rules.
Verb forms with "would" and "will" are less frequently tested on the ACT English section,
but they do occasionally appear. For the ACT, just keep in mind to use "would" in
sentences with past tense verbs and "will" in sentences with present or future
tense verbs.
The construction of verbs with "would" and "will" are "would" + the verb, known as the
conditional tense, or "will" + the verb, known as the future tense. Some examples
include "would run," "would go," and "will talk."
Use the conditional tense to describe things that could occur or things that
haven't yet occurred from the perspective of the past.
Use the future tense to describe things that have not yet occurred or could occur
in the future.
On the ACT, answer choices containing “will have” and “would have” are almost always
incorrect because they tend to cause improper tense switches and make sentences
unnecessarily wordy. The “would have” construction can only be used for something
that could have happened, but didn’t. The “will have” construction describes an action in
the future that will be finished before a second action.
Strategy
I'm not going into more depth because "would" and "will" questions very rarely appear
and this is the only strategy you need to use on the ones I've seen. Most of them simply
require you to pay attention to consistency rules.
Here's one final type of verb form question that can appear on the ACT. It's the least
common, but questions of this type have appeared on previous tests.
Gerunds, infinitives, and participles are all verb forms that don't act like verbs.
Gerunds and infinitives function like nouns. A gerund is formed by adding "ing" to
the end of the verb and an infinitive is formed by adding "to" + the verb.
A participle is a verb that acts like an adjective. Typically, participles end in "ing" or "ed,"
but there are irregularly formed participles.
On the ACT, on rare occasions one of these types of words will be used in the place of
a verb or vice versa.
Strategy
If a verb or one of these "verbs that don’t act like verbs” is underlined, make sure that it
is being used properly. Each sentence must express a complete thought.
The extreme length of this article suggests that writing for an extended period of time.
A. NO CHANGE
B. the manner in which writing
C. that I wrote
D. which had written
Explanation: The use of the gerund, “writing," makes the sentence an incomplete
thought. The sentence needs to express a complete thought. Changing the gerund to a
verb and putting in a subject (the person who did the action) corrects the sentence
fragment without adding an additional clause. The answer is C. All of the other answer
choices are sentence fragments.
Here are some more general rules to keep in mind that will help you correctly answer all
verb questions on the ACT.
#1: If a Verb is Underlined and the Answer Choices are Different Tenses,
Make Sure To Use the Appropriate Form of the Verb
If the answer choices are different tenses of the same verb, then you're probably
answering a question about verb forms. Make sure that the verb follows consistency
rules and the tense is correct.
If the answer choices are different conjugations of a verb in the present tense, you're
most likely dealing with a subject-verb agreement question instead.
#2: Look for Words/Phrases That Indicate Which Verb Tense Should Be
Used
Often, words or phrases elsewhere in the sentence or in surrounding sentences will let
you know what tense to use. If a date in the past is referenced, you should probably use
a form of the past tense. If the word “since” is written, there should probably be a
present perfect verb.
Context clues are placed around the verb to indicate the proper verb tense.
Additional Practice
1. In the next five years, Jessica says that she will become a doctor and she would
buy a house.
A. NO CHANGE
B. buy
C. would have bought
D. might buy
2. As the breakdancer effortlessly contorts his body, he was also smiling at the crowd of
people watching.
A. NO CHANGE
B. has also smiled
C. will also smile
D. is also smiling
3. Last week, Olga participated in a charity event to raise money for autistic children.
She was praised by the organizers of the event for her dedication to the cause.
A. NO CHANGE
B. has been praised
C. will be praised
D. praises
4. My brother claims that he will have become the president of the United States if he is
able to raise enough money to run an effective campaign.
A. NO CHANGE
B. has become
C. became
D. will become
Answers: 1. B, 2. D, 3. A, 4. D
One of these tips is that the ACT prefers shorter sentences. Wordiness and redundancy
are common errors on the ACT English section. The goal for each sentence is to
express the same information in the shortest grammatically correct way.
Explain wordiness.
Explain redundancy.
Show how wordiness and redundancy are tested on the ACT.
Provide actual ACT examples.
Give practice problems to test you on what you've learned.
On the ACT English section, the shortest grammatically correct answer choice that
expresses the same information as the original sentence will be the right answer. Here's
an example of a wordy sentence:
Technically, both sentences are grammatically correct and convey the same
information. However, the second sentence is more concise and less wordy. Check out
another example of a wordy sentence:
After we fix the wordiness error, this is what the sentence looks like:
Again, the meaning of the sentences is the same, but the second sentence is much
more concise and grammatically correct.
Check out these tips for correctly answering wordiness questions on the ACT.
Wordiness Strategy
Approach each question with the mindset that shorter = better. When answering ACT
English questions, start by looking at the shorter answer choices. Keep in mind that the
shorter grammatically correct answer choice must also express the same relevant
information for it to be the right answer.
Plug your answer back into the original sentence to make sure that your answer is
grammatically correct and expresses the same information.
Use these tips to answer the following examples taken from actual ACT English
sections.
A. NO CHANGE
B. in which the magazine where
C. in which
D. being where
Explanation: Even if the original sentence seems correct to you, remember our rule that
shorter is better. Start with the shortest answer choice. That would be answer choice C.
Plug it back into the original sentence. The sentence is still grammatically correct and all
of the relevant info is still there. The phrase “being the place” is unnecessary. The
answer is C.
She worked for thirty years as a teacher and librarian in the field of education in
Baltimore public schools.
Which of the following words or phrases from the preceding sentence is LEAST
necessary and could therefore be deleted?
F. thirty
G. and librarian
H. in the field of education
J. Baltimore public
Explanation: In this type of question, you have to pick which word/phrase can be
deleted without getting rid of any information. Go through the answer choices and see if
the information in the answer choice can be obtained elsewhere in the sentence.
So answer choice F isn’t correct because if we got rid of “thirty”, we would have no way
of knowing that she worked for thirty years. The answer is H. If we got rid of “in the field
of education," we would still know that she worked in the field of education because she
worked in Baltimore public schools. Therefore, the phrase “in the field of education” is
unnecessary.
Are you figuring out how to solve these wordiness questions? Here's a final one for you
to try.
Over many weeks, as time goes by, her collection slowly grows: clay bowls, cups,
vases, and sculptures fill the studio.
A. NO CHANGE
B. with the passing of time,
C. gradually,
D. OMIT the underlined portion
Explanation: Again, let’s start with the shortest answer. That would be answer choice D,
omitting the underlined portion. Is the sentence still grammatically correct? Does it have
the same meaning if we omit “as time goes by?"
Yes!! The underlined phrase is unnecessary because “over many weeks” implies “as
time goes by over many weeks”. Therefore, the underlined phrase is superfluous
and the answer is D.
Another error which can be corrected by implementing our shorter is better rule is
redundancy.
Justin is a very friendly and amicable guy.
Because "friendly" and "amicable" are synonyms, we can shorten the sentence by
getting rid of one of the adjectives that describes Justin. The second way that a
redundancy error is presented on the SAT is that a phrase will be added that is implied
by another word or phrase in the sentence. Take a look at the following example:
This article contains pertinent information that offers relevant facts for doing well in
ACT English.
The words "pertinent" and "relevant" are synonyms. The phrase "that offers relevant
facts" doesn't add any new information to the sentence; therefore, it can be eliminated.
This is the corrected version of the sentence:
This article contains pertinent information for doing well in ACT English.
Does that make sense? The sentence is shorter, grammatically correct, and it
expresses the same information. Here are some tips to help you solve redundancy
questions on the ACT.
Redundancy Strategy
Look at the shortest answer choices first. Plug the shortest answer choice back into
the original sentence. If the sentence maintains its meaning and is grammatically
correct, that's the right answer. Make sure that two synonyms aren't being used to
describe the same thing. Verify that the information in the underlined phrase is not
implied elsewhere in the sentence.
Use these tips to answer the following redundancy questions taken from real ACTs.
F. NO CHANGE
G. perhaps I've
H. I've possibly
J. I've
Explanation: Well, the underlined phrase indicates that we’re probably dealing with a
redundancy question. The words “might” and “possibly” are synonyms, so we don’t
need both of them. Let’s look at the shortest answer choice. That's J. “I’ve” is just the
contraction for “I have” so the only change to the sentence is getting rid of “might
possibly." Will the meaning of the sentence change? No. The word “maybe” indicates
that I “might possibly” have met them. Therefore, “might possibly” is redundant and the
answer is J.
Madonna wants you to have one more chance to correctly answer a redundancy
question.
And just for good measure, here's one final example of a redundancy question.
He also began to calculate annual tables of yearly sets of astronomical data, which
became the basis for almanacs published under his name from 1792 through 1797.
A. NO CHANGE
B. covering a year's worth
C. about twelve months
D. OMIT the underlined portion
Explanation: Can the information in the underlined phrase be obtained elsewhere in the
sentence? Yes. The word “annual” means yearly. If there were annual tables of
astronomical data, then we already know that there were yearly sets. Therefore, the
phrase is unnecessary. The shortest answer choice is D. Will the sentence be
grammatically correct if we omit the underlined portion? Yes. The answer is D.
These questions aren't overly complicated, but you can easily miss a redundancy
question if you don't read each sentence carefully. Make sure to follow the strategies I
gave you for redundancy questions.
Here are some general guidelines to help you correctly answer both wordiness and
redundancy questions.
General ACT English Strategies for Wordiness and
Redundancy
If the shortest answer choice maintains the meaning of the original sentence and is
grammatically correct, then the shortest answer choice will be the right answer. Make
sure that the shortest answer choice is still grammatically correct and expresses the
same information as the original sentence.
Because shorter is better, determine whether the shortest answer choice is appropriate
for the sentence. If a more concise answer choice maintains the meaning of the original
sentence and is grammatically correct, then that is the right answer.
If the information in the underlined word or phrase can be gathered from other words or
phrases in the sentence, then the underlined word or phrase is redundant and should
be omitted.
I've created some realistic practice problems to test your knowledge of wordiness and
redundancy on the ACT.
A. NO CHANGE
B. minimal and not abundant.
C. at a minimum.
D. minimal.
2. In the next two weeks of time in the future, Tarica will be receiving a check for the
legal work she performed for her client.
A. NO CHANGE
3. The teacher became increasingly frustrated with the rowdy student who acted in a
wild, disorderly manner.
A. NO CHANGE
4. George, after thirty years of work in the automotive industry, was thoroughly
overjoyed to retire and no longer go to his job.
A. NO CHANGE
Answers: 1. D, 2. C, 3. D, 4. D
The modifier errors on the ACT English may not be quite as funny as Groucho's, but
you still have to understand how they work. To that end, we'll be covering both the basic
grammatical concepts behind faulty modifiers and how to apply that knowledge on the
test:
There are two ways to correct this kind of error: you can either replace the incorrect
noun (in this case "trees") with the correct one, making any changes necessary to
preserve the meaning of the sentence (fix 1), or you can turn the intro phrase into a
clause that includes the subject the phrase is meant to be describing (fix 2). These
techniques sound more complicated than they are, so let's put them to use remedying
our issue with the running trees:
Fix 1: Running toward the lake, I saw the trees swaying in the wind.
Fix 2: As I ran toward the lake, the trees were swaying in the wind.
Both versions of the sentence are equally correct. You won't be asked to choose
between the two options—instead, which approach makes more sense will depend on
which part of the sentence is underlined and what answer choices you're given.
If only the main clause is underlined, you'll need to pick the answer with the correct
subject—what I refer to above as fix 1. For example:
Incorrect: With tears running down her cheeks, Susie’s sadness was obvious.
Even though the distinction may not seem important, "Susie" and "Susie's sadness"
aren't actually the same, and Susie's sadness can't cry. If the second half of the
sentence is underlined, the best way to correct that issue is to replace the incorrect
noun "Susie's sadness" with the correct one "Susie."
Initiated by a few loud teenagers, the crowd of concert goers quickly joined the heckling
in order to drive the band off the stage.
A. NO CHANGE
B. in order to drive the band off stage, the crowd of concert goers quickly joined the
heckling.
C. the point of the heckling, quickly joined by the crowd of concert goers, was to drive
the band off stage.
D. the heckling, which the crowd of concert goers swiftly joined, was intended to drive
the band off stage.
What's the problem here? That "the crowd" wasn't initiated by the teenagers, the
"heckling" was. As such, "heckling" has to come immediately after the comma, no
matter how tempting the other choices are. D is the correct answer.
On the other hand, if only the introductory phrase is underlined, you have
to convert the introductory phrase into a clause that clarifies who or what is being
described (fix 2). Let's examine our example about Susie from this perspective.
We can't alter the subject in this case, so we need to turn the prepositional phrase into a
dependent clause that makes it clear that the cheeks belong to Susie.
Let's take a look at how this works in a multiple choice question from an official ACT:
As written, this sentence is saying "she" has died down, which doesn't make sense: it
should be the fire. But because only the first part is underlined, we can't change "she" to
"the fire," instead we have to pick an answer that makes clear that the fire is what's
dying down. Clearly, F won't work, and neither does G, since it has the unclear pronoun
"it."
H and J both specify that we're talking about the fire, but "with" in H doesn't make much
sense here—J is the best choice since it maintains the temporal relationship (she bricks
up the firebox after the fire dies down), while clarifying what exactly is dying down.
On the ACT, you may also see dangling modifiers in the context of more general
sentence organization questions. In these cases, all or most of the sentence is
underlined.
There's no single correct approach to these types of questions; instead, you need
to rule out any answers with grammatical issues, and choose the clearest answer
among the remaining options. Let's work through an example from an official ACT
English section:
First we need to figure out who or what is "cheek to cheek," and the only things in this
sentence with cheeks are the dancers. Thus if the sentence starts with "cheek to
cheek," a noun or pronoun indicating the dancers should come immediately after. Both
F and J incorrectly put other nouns in that position (though "cheek to cheek" describe
how the couples dance, the dance itself isn't "cheek-to-cheek").
H switches "cheek to cheek" and "relaxed two-step tempo," which actually makes even
less sense, since the latter phrase isn't a modifier (it's a noun phrase). This leaves G,
which orders the modifiers logically to indicate that they describe how the couples are
dancing.
Let's revisit the key rule for modifier placement: a modifier must be next to
whatever it's modifying. More specifically, single word modifiers, i.e. adverbs and
adjectives, generally go before the word they're modifying, while prepositional
phrases go after what they're modifying.
"Weird" is an adjective, which means it can be used to describe a noun or pronoun. The
only one of those in the sentence is "monster," so "weird" should go before monster. (In
some cases, this kind of construction might indicate an adverb vs. adjective error.)
Incorrect: The monster sent long letters to his new friend full of details about his life.
Correct: The monster sent long letters full of details about his life to his new friend.
In the first version of this sentence, the phrase "full of details about his life" comes after,
and is thus modifying, "friend." But it's meant to describe "letters." The corrected
sentence places the phrases in the appropriate order.
There are exceptions to these guidelines, but the idea that modifiers should be next to
what they modify is a good rule of thumb. Let's look at a sample ACT style question:
Edgar Allan Poe is revered for his frightening and stylish stories by literary critics, many
of which appear in high school text books.
A. NO CHANGE
D. by literary critics who deem his stories frightening and stylish,
At first glance, there's nothing much wrong with this construction: sure, the order of the
prepositional phrases "for his frightening and stylish stories" and "by literary critics" is a
bit weird, but the ACT isn't usually that nitpicky.
There's actually a bigger issue here, with the non-underlined clause "many of which
appear in high school text books." What's which referring to? The stories, not the critics.
As such, we need to reorder the underlined portion to put "stories" at the end, making B
the correct choice.
Though this style of question appears fairly frequently on the ACT, there's another style
of misplaced modifier question that you are almost certain to see at least once on the
test. These questions ask where a modifier should be placed or where it would NOT be
acceptable. In approaching these questions, you want to keep the basic rules in mind
as you try the phrase or word out in different spots.
The first key to this question is catching the EXCEPT: we're looking for the answer
choice that doesn't work. I've written out each of the four choices below—can you spot
the one that doesn't work?
B. In the 1700s, the O'odham first encountered the guitars of Spanish missionaries.
C. The O'odham first encountered the guitars in the 1700s of Spanish missionaries.
D. The O'odham first encountered the guitars of Spanish missionaries in the 1700s.
If you guessed C, good job! Putting "in the 1700s" after guitars interrupts the noun
phrase "guitars of the Spanish missionaries"—all of a sudden the 1700s belong to
Spanish missionaries. A is fine because we're talking about what the O'odham did in the
1700s, and in both B and D the phrase is modifying the whole sentence, which makes
sense since it's describing when this event occurred.
What to watch for:
Make sure to watch out for EXCEPT and NOT. Even though they're in all caps,
they can be easy to skip over—get in the habit of circling them every time you
see them
Think about both what a modifier is currently describing and what it's actually
meant to be describing
Watch out for answers that fix the original modifier issue but are ungrammatical
in another way and those that create modifier errors when attempting to rephrase
a sentence
Remember that there are a lot of different ways to fix faulty modifiers—use
process of elimination to narrow down wrong answers rather than focusing on
one specific way of correcting the error
A. NO CHANGE
2. I was waiting in line with a young girl who was sketching in a flowing scarf with
colored pencils.
F. where it is now.
G. after the word line.
3. Suggested by me the new mystery novel, supposedly a page turner, our book club
decided to read it unanimously.
A. NO CHANGE
B. Our book club unanimously decided to read the new mystery novel I suggested,
which was supposed to be a page turner.
C. A page turner, unanimously the new mystery novel was decided by our book club,
suggested by me.
D. Decided on by our book club, the new mystery novel, a page turner, was suggested
unanimously by me.
Answers: 1. C, 2. H, 3. B
Idiom questions on the ACT are different from the other grammar questions on the
ACT. Why? Idiom questions don’t conform to specific rules. You have to rely on your
intuitive grasp of English and your familiarity with certain phrases.
Because you’re likely to encounter at least a few idiom questions on the ACT English
test, I’ll provide you with some information about idioms that should help you raise your
ACT English score.
What Is an Idiom?
Idioms are phrases or expressions that do not conform to simple rules. Each idiom,
by definition, is unique. Most people think of idioms as expressions that often have
figurative meanings different from their literal meanings. Examples of this type of idiom
include "actions speak louder than words," "barking up the wrong tree," and "make a
long story short." However, the ACT does not test you on these colloquial expressions.
ACT English idiom questions will test you on different types of idioms.
Prepositional Idioms
For prepositional idioms, you must know which prepositions to use with a given word
based on the context of the sentence. For example, you should say that you
"wonder about" something, not "wonder on" something. You're "suspicious of"
something, not "suspicious by" something. There is no rule to determine the correct
preposition to use. You must be familiar with the phrase or rely on what you think
"sounds right." Here's an example sentence with a prepositional idiom:
Unsurprisingly, after Corey decided not to pay back the substantial loan given to him
by his best friend, everyone was outraged for Corey’s behavior.
This is a typical sentence with an idiom error you may encounter on the ACT. There's
no violation of a specific grammar rule in the sentence. However, the phrase "outraged
for" is incorrect. The sentence should read:
Unsurprisingly, after Corey decided not to pay back the substantial loan given to him
by his best friend, everyone was outraged by Corey’s behavior.
The proper expression is "outraged by." There is no rule that allows you to determine
the correct idiomatic expression. Familiarity with the given phrases will greatly help you
in identifying idiom errors.
Gerunds are verbs that are used as nouns and end in "ing." Examples of gerunds
include skipping, talking, and performing. Infinitives are verbs used as nouns and are
constructed by using the word "to" plus a verb. Examples of infinitives include to do, to
analyze, and to explain.
So what are some examples of idioms with gerunds or infinitives? The correct phrase is
"capable of being," not "capable to be." The proper idiomatic expression is
"regarded as being," not "regarded to being." For these types of idioms, you need to
know which preposition to use and whether to use a gerund or an infinitive.
With some idioms, depending on the context, it's acceptable to use an infinitive or a
gerund. Here's an example with the gerund in bold:
I neglected doing my homework.
The phrase "succeed in to graduate" is incorrect. Do you know the right idiomatic
expression to use? This is the corrected version of the sentence:
There's no rule to learn that will indicate that "succeed in graduating" is the correct
phrase. Here's one more example of an idiom error:
This is how the sentence looks after we fix the idiom error:
The infinitive form should be used with the word "decides." Now here are couple of
actual idiom questions from real ACTs.
A. NO CHANGE
B. of wearing
C. to wear
D. wearing
Explanation: The correct expression is “right to wear clothing of his or her own
choosing." The answer is C.
Furthermore, ESL students are less likely to be able to identify idiom errors. Knowledge
of idioms tends to be built throughout a lifetime of exposure to them.
Idiom questions are the only ones where solely relying on what "sounds right" is
likely to give you the right answer. These questions do not require you to understand
and apply a rule. If you're familiar with the specific idioms that appear on the ACT
English section, you can easily spot any idiom errors.
#2: The question may be testing idioms if the answer choices are all
prepositions.
#4: Review and familiarize yourself with the list of idioms below.
It's not practical for you to memorize every single idiom on this list. There are
usually less than a handful of idiom questions on the ACT English section. Spending
numerous hours learning hundreds of idioms would not be the best use of your study
time.
come to a conclusion
come to a halt
come to an end
in the dusk
modeled on
principles of
such as
PREPOSITIONAL IDIOMS
About
anxious about
ask about
bring about
curious about
hear about
think about
talk about
worry about
Against
advise against
argue against
count against
decide against
defend against
go against
rebel against
As
celebrate as
regard as
see as
view as
At
aim at
arrive at
laugh at
look at
succeed at
By
accompanied by
amazed by
confused by
followed by
go by
impressed by
organized by
struck by
For
advocate for
ask for
blame for
famous for
known for
last for
meant for
named for
necessary for
pay for
ready for
responsible for
tolerance for
strive for
wait for
watch for
From
abstain from
different from
excuse from
far from
obvious from
protect from
Into
enter into
look into
inquire into
read into
In
engage in
fall in love
in A as in B
interested in
succeed in
take in
On
base on
draw on
focus on
impose on
insist on
move on
prey on
rely on
Over
argue over
rule over
talk over
think over
Of
approve of
capable of
certain of
characteristic of
combination of A and B
cure of
deprive of
die of
a fan of
in danger of
in the hope of
in recognition of
made up of
a model of
an offer of
on the border of
remind of
a selection of
a source of
suspicious of
take advantage of
an understanding of
a wealth of
To
able to
accustomed to
adapt to
adhere to
admit to
adjacent to
agree to
as opposed to
belong to
central to
come to
contribute to
devoted to
in addition to
in contrast to
listen to
object to
prefer A to B
partial to
reluctant to
reply to
see to
similar to
a threat to
unique to
With
agree with
bargain with
correlate with
familiar with
identify with
in keeping with
interfere with
sympathize with
trust with
accuse of
admire for
allow
appreciate
capable of
complete
concentrate on
confess to
consider
delay
describe
discourage from
discuss
dislike
effective at
enjoy
escape
finish
forbid
imagine
insist on
permit
plan on
postpone
refrain from
report
resent
resume
stop
tolerate
Prepositions Followed by a Gerund
before
after
without
agree
attempt
choose
condescend
dare
decide
deserve
encourage
expect
fail
intend
love
mean
neglect
offer
plan
prepare
promise
refuse
scramble
seem
strive
swear
tend
threaten
want
A. NO CHANGE
B. stayed
C. to stay
D. is staying
A. NO CHANGE
B. to eat
C. eating
D. of eating
3. Even though I tend to not enjoy magic, I was amazed to the trick the illusionist did at
the end of his show.
A. NO CHANGE
B. by
C. from
D. into
A. NO CHANGE
B. insisted by
C. insisted in
D. insisted to
Answers: 1. C, 2. D, 3. B, 4. A
You may think that of all the grammatical concepts you will encounter on the ACT
English, recognizing a correct sentence will be one of the easiest ones. But did you
know that this is actually one of the most commonly-tested subjects because it can
be very tricky?
Do you know what you need to have in order to have a complete sentence? Can you
reliably tell the difference between a subordinate clause and an independent clause?
Do you know how to use semicolons and conjunctive adverbs?
Read this guide to see how the ACT manages to trick so many students with this
seemingly easy concept.
In this guide I will show you:
Test Yourself
Can you recognize which of the following are correct sentences, and which are
fragments and run-ons?
What Is a Sentence?
You may have heard in your English class that a sentence needs to have a subject and
a verb. More specifically, a correct, complete sentence has to have at least
one independent clause.
The ACT will not test you on any of the grammatical terms we cover, but understanding
the ideas is important. A simple sentence made of one independent clause has three
main characteristics:
1. A subject (a person or thing that is doing an action)
Let's look at one of the above sentences and see if it matches this description.
My dog barks.
There are some rare circumstances in which you will see a complete, correct sentence
where you cannot as easily pick out the subject: direct commands.
You now know everything you need to know about simple sentences! But did you know
that sentences can also be compound or complex? Let's go over these concepts next
to see how the ACT might try to trick you.
Compound Sentences
A compound sentence is a sentence that has more than one independent clause -
meaning two subjects and two verbs.
This kind of construction can start to get tricky, because you have to make sure that
they're joined together correctly. If they aren't, they're called run-on sentences. We'll
discuss these more below.
There are several different ways that you can join independent clauses together
correctly to make a compound sentence.
Coordinating conjunctions are probably the conjunctions you're most familiar with,
including and, but, and or. You can remember all of them with the acronym FANBOYS:
F for
A and
N nor
B but
O or
Y yet
S so
Ben and Kate always enjoyed eating Chinese food. They began to prefer Japanese
food after traveling to Japan.
Here is how you could join these two independent clauses with a coordinating
conjunction:
Ben and Kate always enjoyed eating Chinese food, but they began to prefer
Japanese food after traveling to Japan.
Remember that when using a coordinating conjunction to join two sentences, you
must always use a comma.
Ben and Kate always enjoyed eating Chinese food; they began to prefer Japanese
food after traveling to Japan.
Join the sentences with a semicolon (or period) and conjunctive adverb.
There are several different conjunctive adverbs, but some of the most common ones
are: however, nevertheless, therefore, moreover, and consequently.
You can pair one of these words with a semicolon or a period to show the relationship
between the two sentences you are joining.
However and nevertheless show a contrast.
Ben and Kate always enjoyed eating Chinese food; however, they began to prefer
Japanese food after traveling to Japan.
Remember when using this construction that you should always have a comma after
the conjunctive adverb.
We will talk more about dependent clauses in a moment, but this is how this tactic
would look:
Though Ben and Kate had always enjoyed eating Chinese food, they began to
prefer Japanese food after traveling to Japan.
Once again, note that the dependent clause is separated from the main clause by a
comma.
You will also often find that you need to add a word or two when using this method in
order to have the sentence make more sense, or rearrange the order of the words.
Because of the many variables involved, this approach can be one of the more difficult
options for joining two sentences together.
Let's look more into how dependent clauses are formed and how they are used.
Dependent clauses will begin with a subordinating conjunction that gives you a signal
that it cannot stand on its own. For more on subordinating conjunctions, see this article.
In contrast, look at the dependent clause, "Although he loved his country." This gives
background information for the independent clause that follows, but it does not make
sense on its own. The reader is left wondering what did he do?
Remember that if you encounter a dependent clause on its own on the ACT, it will
always be incorrect. I will discuss this in more detail below in the "Fragments" section.
Sentences with Prepositional Phrases, Appositives,
and Relative Clauses
Prepositional Phrases
You can find a list of the most common prepositions in the English language in this
article. A prepositional phrase consists of (at least) a preposition and a noun that's
called the object of the preposition. Let's look at some examples of how these are used:
The young man in that coffee shop enjoys eating cake. (Prepositional phrase tells
where man is)
The young man enjoys eating cake in that coffee shop. (Prepositional phrase tells
where he likes to eat cake.
Remember that you should always be able to completely delete a prepositional phrase
and still have a full sentence left over. If you don't, then your sentence is incorrect!
Relative Clauses
The young man, who was sitting in the cafe and eating cake, dreamed of getting a
PhD.
In this sentence, the subject of the relative clause is "who." The verb is "was sitting."
Relative clauses can be completely removed from the sentence and you'll still have a
complete sentence left over.
Appositives
Jeff, the young man, sat in the cafe and ate cake while he studied.
In this sentence, "the young man" is an appositive fodescribing the noun "Jeff."
As you may have guessed, you should be able to remove an appositive and have a
complete sentence left over.
What Is a Fragment?
A fragment is an incomplete sentence. There are 6 main mistakes that can make a
sentence a fragment:
How to recognize these fragments: ask what the subject of the sentence is doing.
If you can't answer that question, it's most likely because there isn't a verb!
How to fix these fragments: you must add a verb to show an action or a state of
being.
Sentences with an -ing verb or non-past tense -ed verb and no helping
verb
You can also fix this type of fragment by using the -ed or -ing word as a an adjective
(called a participle) and adding another verb.
The children watching television laughed.
You may notice that some of these examples have multiple issues, like the ones
mentioned above - in that case, it's important to make sure that you have fixed all of the
errors in the sentence.
Thinking about buying a gift for the wedding. Who was thinking about it?
To fix these fragments, add a subject and make sure the verb is present in a correct
form.
The following words are common subordinating conjunctions. If the sentence begins
with one of these, make sure it's attached to an independent clause. If not, it's a
fragment!
You can also fix them by getting rid of the subordinating conjunction. This turns
them into independent clauses that can stand on their own.
He was late.
Added Detail Fragments
How to recognize these fragments: look for words like "such as," "including," and
"for example" that start a sentence, but explain something in a previous
sentence. If this kind of construction isn't attached to an independent clause with
a subject and a verb, then it's a fragment.
I know many different dances, such as: the salsa, the polka, and the waltz.
In order to fix these fragments, add the detail fragment to the main sentence it is
describing.
He likes exploring new places, such as old towns and big cities.
You can also turn the fragment into a complete sentence by adding a subject and
a verb:
She likes a lot of different kinds of movies. For example, she enjoys both horror and
comedies.
Finally, make sure that anything that comes before a colon can stand on its own
as a sentence:
I know many different dances: the salsa, the polka, and the waltz.
How to fix these fragments: Add a subject or verb as needed. Make sure that if you
eliminate the prepositional phrase or non-essential clause, that there is a complete
sentence left.
James, my cousin, ate the whole pie. → James, my cousin, ate the whole pie.
CORRECT
The girl, who was the best artist in the class, won another award. → The girl, who was
the best artist in the class, won another award. CORRECT
What Is a Run-on?
A run-on is when two or more sentences run together and are not separated by the
correct punctuation. There are three main types of run-ons:
Comma splices
Fused sentences
Sentences joined by a conjunctive adverb and commas
Comma Splices
A comma splice is when two complete independent clauses (full sentences) have
been joined together by a comma and nothing else. A comma can never join
together two complete sentences on its own.
He had worked for the government for several years, he now wanted to try a job in a
small start-up company.
How to recognize these run-ons: when you see a sentence with a comma in the middle,
check to see if there is a coordinating conjunction (see this article for more info) that
joins the sentences together, or if one of the clauses is a subordinate clause that
begins with a subordinating conjunction. If not, the sentence is a run-on.
Fused sentences
A fused sentence is when two or more sentences run right into each other, with no
punctuation at all.
He had worked for the government for several years he now wanted to try a job in a
small start-up company.
How to recognize these run-ons: look to see if you have two unconnected subjects and
verbs, with no indication of more than one independent clause (see above the correct
ways to join two independent clauses).
The final type of run-on is a bit trickier. Conjunctive adverbs are used with periods or
semicolons to connect two independent clauses, so if they're paired with two commas
instead, it creates a run-on:
He had worked for the government for several years, however, now he wanted to try
a job in a small start-up company.
However, conjunctive adverbs that are connecting two complete sentences punctuated
with a period or semicolon are sometimes moved into a sentence. This is perfectly
acceptable:
He had worked for the government for several years. Now, however, he wanted to try
a job in a small start-up company.
How to recognize these run-ons: when you see a conjunctive adverb (however,
moreover, nevertheless, therefore, consequently, etc.) with commas on either side of
it, make sure it's being used to transition from a previous sentence and not to connect
two independent clauses.
To fix run-ons
...you will need to use the rules outlined in the first section to correctly join two
sentences.
He had worked for the government for several years. He now wanted to try a job in a
small start-up company.
He had worked for the government for several years, but he now wanted to try a job
in a small start-up company.
He had worked for the government for several years; he now wanted to try a job in a
small start-up company.
He had worked for the government for several years; however, he now wanted to try
a job in a small start-up company.
Although he had worked for the government for several years, he now wanted to try
a job in a small start-up company.
#6: If the two clauses have the same subject, re-write as a sentence with one
subject and two verbs.
Note that when you fix a sentence this way, you should not use a comma with your
FANBOYS conjunction.
He had worked for the government for several years but now wanted to try a job in a
small start-up company.
Specific Strategies for Fragment and Run-
on Questions
As you're reading through the different passages on ACT English, if you come across a
question that you think may be about fragments or run-ons, use the following steps:
Step 1
Carefully read the prompt if there is one. Sometimes the ACT will ask you to correct an
error, and other times they will ask you to find the only answer that does NOT work.
Step 2
After reading the sentence, does a mistake jump out at you? If not, look for the
following telltale signs of a fragment or run-on:
According to the ACT, not all mistakes are created equal. There are a few concepts
they like to test more than others, and they have some obvious markers.
If the underlined section has a colon or semicolon, be on the lookout for a fragment
on either side.
If the underlined section has a comma, it will very likely be a comma splice.
Immediately cross out any answer choices that make the above errors.
Step 3
If you are still uncertain about the correct answer: you can immediately eliminate
anything that is grammatically identical. For example, a semicolon (;) is
grammatically identical to a period and a new sentence. If one is incorrect, they will both
be incorrect.
Step 4
Select a final answer choice that follows the above rules and also follows the standard
style rules used on ACT English:
A. read; he
B. read, and he also
C. read he
D. read. He
From reading the prompt, I can see that the given sentence is correct as is, and so
are three of the answer choices. This is a “backwards” question that asks us to find
the only incorrect answer instead of the only correct one.
I can see that there is a comma in the underlined portion, so I need to see how it’s being
used. It’s pretty obvious that the comma is being used together with the FANBOYS
conjunction “and." This is one of the correct ways of joining two independent clauses,
as detailed above.
From double checking I can see that I have two independent clauses with their own
subjects and verbs: “Benjamin’s grandmother taught him to read” and “he attended a
one-room Quaker school when the farmwork slowed down during the winter.”
So which of the answer options also show correct ways to join two independent
clauses?
(A) has the independent clauses joined by a semicolon on its own. This is a correct
way to join two independent clauses, so (A) is not the answer.
(B) joins the two independent clauses together in the same way that the original
sentence does, with a comma and a FANBOYS conjunction. The only thing that is
different is that it adds an extra adverb, “also," which is unnecessary but not incorrect.
(C) does not do anything to join the two clauses together - this is a classic
example of a “fused” run-on sentence. Therefore, (C) is the correct answer as it is
the only option that does not join the clauses together correctly.
(D) separates the two independent clauses with a period, which is another correct way
of dividing two independent clauses, so (D) is not the answer.
A. NO CHANGE
B. now" that
C. now." This
D. now,"
2. The product manufacturer begins by creating a flexible steel grate. Across each
opening, a wire mesh, evenly perforated, able to accommodate exactly one hook.
A. NO CHANGE
B. perforated, by being able
C. perforated, which is able
D. perforated, is able
A. NO CHANGE
B. dance in which it is performed
C. dance, performing
D. dance, performed
4. Around this time, the Glagolitic alphabet introduced by St. Cyril; became further
integrated into the culture.
A. NO CHANGE
B. St. Cyril
C. St. Cyril:
D. St. Cyril,
5. To ensure the safety of the drug, the pharmaceutical company had it tested in several
trials, however, they found no evidence of averse side effects.
A. NO CHANGE
B. which
C. who
D. he
6. Peter the Great, who founded St. Petersburg to be Russia's "window to the West"
and indirectly named the city after himself.
A. NO CHANGE
B. West," indirectly naming
C. West," indirectly named
D. West" and
Answers: 1. C; 2. D; 3. D; 4. B; 5. B; 6. C
In high school, I had a history teacher who was known for going off on tangents—he
would start giving us his lesson on the French Revolution but end up telling us all about
the Peregrine falcon. These facts were interesting, but they were hardly relevant to the
issue at hand (how Marie Antoinette got her head chopped off).
Similarly, ACT English tests your ability to spot places where a passage veers off
topic. Of course, it does so in it's own special way, which this post will cover! Here's
what you'll need to understand to approach relevance questions with confidence:
Keep in mind that relevance questions are very similar to redundancy questions, which
are much more common. The two types of question operate on the same basic principle
—cut anything you don't need—but relevance questions are about whether
information adds to the point of the paragraph, while redundancy questions are about
whether a word or phrase is repeating something that's already been said.
There's nothing obviously wrong with this sentence, so we need to look at how the
answer choices are different from each other: they all add information about the Navajo.
Given that the information provided by the three answer choices is basically the same
but the phrasing is different, it would be easy to conclude that this is a wordiness
question. However, we first need to determine whether the extra information is relevant.
The additional phrases in answers B, C, and D all describe the size of the Navajo—this
information isn't given anywhere else, so it isn't redundant, but that doesn't mean it's
necessary.
The size of the Navajo has nothing to do with their creation of a linguistic code, which is
what the paragraph is about. As such, this information is irrelevant and shouldn't be
included. A is the correct answer.
So how do you determine if a piece of information is relevant? There are two issues to
consider:
If the answer to the first question is no, then don't even worry about the second
one—the information isn't relevant and should be omitted. (This principle applies
to most of the relevance questions on the ACT, including the example above.)
Test items that do require you to consider the second question are a bit more
challenging. If a piece of information seems like it might be relevant, consider
whether it's necessary for your understanding of the passage. Does it clarify a
previous point or introduce a key detail? If not, it probably isn't relevant.
This concept may seem complicated, but it's actually not too bad. Let's look at
an example of relevant versus irrelevant information:
The first underlined section adds information, but the actor who played da Vinci in a
movie is not related to the fact he was a Renaissance man (someone who has great
expertise across a variety of topics). The underlined portion of the second sentence, on
the other hand, provides information that clarifies why da Vinci is a famous Renaissance
man.
Julia wanted to move into her brother's room after he graduated from high school, but
she wasn't able to because he ended up living at home for his first year of college.
A. NO CHANGE
D. OMIT the underlined portion, ending the sentence with a period after to.
At fist glance, this question might appear to be a wordiness question, since choice B
shortens the original underlined portion. However, if you look closely at the different
answers you'll see that they each provide different information—this fact makes it a
relevance question.
Our next step is to summarize the main idea of the sentence: Julia wanted to take her
brother's room but couldn't because he was still living in it. We can eliminate choice C,
since the brother's choice of major is clearly not relevant to this topic.
The remaining choices involve deciding how much of the information in the original
version is relevant: all of it, the first half of it, or none of it.
Let's start by deciding whether we should omit the whole underlined portion. What does
this section of the sentence tell us? Why Julia can't have the room. The sentence would
still make grammatical sense without this part, but it would be lacking a key piece of
information, so we can eliminate D.
Finally, we have to choose between A and B. The correct choice is A because the
phrase "for his first year of college" connects back to the fact that Julia was planning on
taking her brother's room after he finished high school and helps to explain why she
wasn't able to do so.
Most ACT English relevance questions will be much simpler than this one, but the
process you use to answer them is the same.
It's important to keep in mind that even though most of the time the extra information
will be irrelevant and need to be cut out, it will sometimes be necessary, so you
always have to read the information carefully and decide.
#1: Watch for underlined sections where some of the answers include extra
descriptive information.
#2: Omit information that isn't related to the main idea of the sentence or
paragraph.
1. Because owls rely on their wings to hunt, wing injuries are major issues for the birds,
which are nocturnal.
A. NO CHANGE
C. the birds.
F. NO CHANGE
3. Holi is an Hindu festival during which revelers throw colored dye into the air. Yom
Kippur is the Jewish New Year. This practice, which results in participants skin and
clothes being temporarily colored, has been adopted in the US for large parties and fun
runs.
A. NO CHANGE
Answers: 1. C, 2. F, 3. D
Author Technique on ACT English: Passage
Strategy
Read this article for a thorough explanation of author technique and guidance on how to
figure out author technique questions.
In this post, I'll do the following:
A portion of a sentence will be underlined, and you have to determine if the given
phrase or the other answer choices will best fulfill the purpose given in the question.
This type of question requires you to analyze phrases and determine whether they
would satisfy the stated goal.
Now, let's look at examples of author technique questions from actual ACTs.
Example #1
F. NO CHANGE
G. the fire is stronger than ever
H. there is more heat being produced
J. a kind of intense blaze takes place
Example #2
As the potter takes bricks away to create an opening into the oven, an expanding
view of gleaming shapes rewards the artist for months of hard work.
The writer would like to suggest the potter’s cautious pace and sense of anticipation in
opening the kiln. Given that all the choices are true, which one best accomplishes the
writer’s goal?
F. NO CHANGE
G. removes bricks by hand
H. removes one brick at a time
J. experiences great anticipation and removes bricks
Example #3
Given that all of the choices are true, which one best develops the paragraph’s focus on
the roles that letters played in Emily Dickinson’s life?
A. NO CHANGE
B. Her personal interests also included keen observation of the natural world around
her.
C. Though she produced volumes of letters, none were shared publicly until after her
death.
D. She enjoyed hearing their news and reflecting with them on political events.
Strategy for Author Technique Questions
Example #1
The writer would like to indicate that at this point the fire is extremely intense. Given that
all the choices are true, which one best accomplishes the writer’s goal?
F. NO CHANGE
G. the fire is stronger than ever
H. there is more heat being produced
J. a kind of intense blaze takes place
The question is asking which answer choice does the best job of indicating that the fire
is extremely intense. The correct answer choice will make it obvious to the reader that
the fire is extremely intense.
#2: Check the Answer Choices to See if They Acomplish the Intended
Goal
For our example, go through each answer choice and determine if it indicates that the
fire is extremely intense. The answer choice has to match the goal as closely as
possible. The fire can’t be somewhat intense. We’re looking for extremely intense.
First, look at the original sentence. Does “a controlled inferno roars” accomplish the goal
of showing that the fire is extremely intense? Yes. An inferno is a large, intense fire. And
the verb “roars” further indicates that the fire is extremely intense.
However, go through the rest of the answer choices quickly just to make sure there isn’t
a better answer.
Answer choice G, “the fire is stronger than ever,” indicates that the fire is more intense
than before, but it doesn’t directly state that the fire is extremely intense.
Answer choice H, “there is more heat being produced,” does not even reference the
intensity of the fire.
Answer choice J, “a kind of intense blaze takes place,” is the trap answer. It has the
word “intense” in the phrase, but we’re looking for a phrase that shows that the fire is
extremely intense. A “kind of intense” fire is not extremely intense.
Usually, it’s easier to eliminate the obviously wrong answer choices first before
eventually arriving at the right answer. For our example, H could be eliminated right
away since there isn't even a reference to the fire. Then, after looking more closely at
the other answer choices, you should be able to eliminate G and J.
Not only should the answer choice fulfill the purpose of the author, but the resulting
sentence should be grammatically correct and match the essay’s tone.
Typically, sentences in ACT English are moderately formal. They’re not extremely
formal or very casual.
Also, the ACT stresses that sentences should be as concise as possible.
For our example, we didn’t really need to consider tone and conciseness, but if you
were considering answer choice F, “a kind of intense blaze takes place”,
its construction makes the phrase sound awkward and wordy. You could convey the
same meaning more succinctly.
Going through all of these steps should take roughly 15-30 seconds.
We'll go through this process again with another question from a real ACT.
At one point, Emily sent a draft of her poem “Safe in Their Alabaster Chambers”
to Susan, who read the poem.
Given that all the choices are true, which one would most clearly describe an interaction
between Susan and Emily during Emily’s writing process?
A. NO CHANGE
B. liked the poem tremendously.
C. considered and thought about the poem.
D. praised the poem but suggested revisions.
First, we determine what the question is asking. We need to choose a phrase that
suggests interaction between Susan and Emily during Emily’s writing process.
Therefore, if a phrase doesn’t show interaction between the two, it can be eliminated.
The original phrase, “read the poem," doesn't show interaction. If Susan “liked the poem
tremendously," that doesn't show interaction between them either, so we can eliminate
B. If Susan “considered and thought about the poem," that doesn't demonstrate any
interaction so answer choice C can be eliminated as well.
However, if Susan “praised the poem but suggested revisions”, that does describe an
interaction during the writing process. If she “suggested revisions,” Susan told Emily
how to change the poem to make it better. That's an interaction during the writing
process. The answer is D.
Quick Review of General Strategies for Author
Technique Questions
Take a closer look at the second half of my first sentence: "people often misuse
pronouns, especially when you speak." Who is "you" referring to here: "people."
However, the correct pronoun for the third person plural is "they." The correct version of
the sentence is "people often misuse pronouns, especially when they speak."
On the ACT English section, you'll be expected to spot these kind of errors in a variety
of different contexts. I'll go over both the basic rules for pronouns and the common
mistakes you'll see on the test, so that you can approach the test with confidence.
Feature image credit: Chiltepinster (Own work) [CC BY-SA 3.0], via Wikimedia Commons
What's a Pronoun?
A pronoun is a word that takes the place of a noun—that's it! Usually we think of
pronouns as words like I, him, they, or its, all of which are definitely pronouns. But so
are words like everyone, which, and each. Any word that replaces a noun is a pronoun.
For more detailed information on pronouns, take a look at our guide to parts of speech.
What's an Antecedent?
An antecedent is the noun a pronoun replaces. (You may also have heard these
called referents.) Let's look at an example—the pronoun is in bold and the antecedent is
underlined:
No matter how many different pronouns there are in a sentence, each one must have a
clear antecedent.
Again, though this concept may sound confusing, it's actually pretty intuitive. Let's revisit
our example from above, with a slight tweak:
As a review, the most common pronouns and their correct uses are listed below:
The key idea here is that pronouns must always have a clear antecedent and agree
with the noun that they replace. Now that we've established what that means, let's
look at how these errors appear on the ACT English.
Pronoun Disagreement in Number
I mentioned above that plural nouns require plural pronouns and singular nouns
require singular pronouns. Usually, that rule feels pretty obvious: it doesn't make
much sense to refer to Jim as "them" or the Avengers as "it." However, we're talking
about the ACT, so the errors are going to be a little harder to spot. Let's go over some of
the trickiest cases and how you can catch them.
But in the context of a more complex sentence, or an entire paragraph, these errors get
much harder to spot:
she = Clara
her = Clara's
they = the library
them = the novels
Do you see the problem? The library is singular and can't be referred to as "they." As
such, the sentence should read:
Please have your child bring their crayons to school with them.
Sounds fine, right? But take a closer look. "Their" is taking the place of "your child's" in
this sentence, but "their" is a plural pronoun and "your friend" is only one person.
Using "their" seems right because we the child could be a girl or boy and there is no
singular non-gendered pronoun for people in English. However, you can't use 3rd
person plural pronouns (they, their, theirs, or them) to refer to singular people of
unclear gender.
Let's work through an official ACT question, so you can see how these concepts work in
context.
Let's start by finding the antecedent. So what's covered? "Most kayaks," which is plural,
so we immediately know A can't be correct.
The next step is eliminating distractors. Pronoun questions on the ACT can be tricky
because the test writers tend to include answer choices that seem reasonable but
couldn't possible work, confusing what the question is actually about. Here those
choices are B and D: when used as a pronoun "one" refers to a person and "which"
should come after a comma, not a period (it creates a dependent clause).
This leaves only C, "They are," which is the right answer since "they" is the correct
pronoun to stand in for "kayaks."
Pronoun Disagreement in Person
Pronoun agreement in person is a slightly weirder concept than agreement in number,
but it's usually pretty easy to spot. The basic concept is that a pronoun must reflect
the type of thing it's replacing. This fact means that you must use "it" to replace "the
sour candy" but "she" to replace "Jennifer Lawrence."
As with agreement in number, the first step is always to make sure that you know what
a pronoun's antecedent is and that the pronoun and antecedent agree. Beyond that
basic principle, avoiding issues with agreement in person is mostly a question of
consistency—if you start a sentence in the first person, you shouldn't suddenly
switch to the third person without a clear reason.
On the ACT, pronoun errors with agreement in person are usually related to consistency
—if a passage is in the third person you can't inexplicably introduce the first person
halfway through and vice versa.
The most common such issue is with "one" and "you." Both are equally correct to refer
to a non-specific individual—"you" is simply less formal than "one"—but a passage
should stick to one or the other. (Like this article does!)
The incorrect sentence uses two different pronouns, while the corrected versions stick
to one or the other.
One of the most common mistakes students make on the ACT English is assuming that
the more formal choice, in this case "one," is necessarily better. However, it's far more
important to be consistent with the passage as written and to make sure pronouns
match their antecedents.
First, we find the antecedent. Who or what is hearing the songs? Elsewhere in the
paragraph, it's "I." Our primary goal with agreement in person is to remain consistent, so
it should be the same here—F is the correct choice.
G and H might be tempting, but you shouldn't pick an answer that introduces a new
pronoun without a clear antecedent. J can't be correct because it creates a sentence
fragment.
Unclear Antecedents
We've covered the basic rules for ensuring that pronouns agree with their antecedents,
but what do you do when a pronoun doesn't have an antecedent? It's important to
remember that a pronoun's antecedent must be a noun and you must be able to
circle it in the text. In other words, it's not enough for you to know what the pronoun is
referring to, the antecedent also has to be obvious in the text.
In order to get the concert tickets from the box office, I need to know when they'll be
there.
Who does "they" refer to? Presumably, the people who work in the box office, but the
noun "employees" doesn't appear, so there's no clear antecedent. A correct version
might read:
This construction may be fine in spoken English, but it would be absolutely incorrect on
the ACT. Thought it's clear what Luigi is reacting to, there's no actual noun in the text
that is serving as an antecedent for that.
This, that, and these and those must have clear noun antecedents, just like any
other pronoun. However, it's fine for the noun to come immediately after the pronoun.
A correct version of the above would be:
Luigi: That move is so annoying.
If this, these or those are underlined, then the question is likely about this type of
error. (That being underlined can mean there's an antecedent issue as well, but it's
more commonly a relative pronoun question.)
As with any pronoun question, the first step here is to determine the antecedent. Since
an antecedent must be a noun that appears in the text, our options are "Banneker," "his
studies," and "the paths of the Sun, Moon, and other celestial bodies." However, none
of these nouns are things that can be used to predict a solar eclipse—the sentence is
trying to say that he used his calculations of the paths to make the prediction. The only
answer that makes that fact clear is G.
What to watch for:
A. NO CHANGE
B. he
C. the people
D. she
2. You may not want to go looking for the cookbook, but to ensure there are pancakes
at breakfast, one must find the recipe.
F. NO CHANGE
G. you must
H. they must
J. must
3. One of the books was full of fairy tales, but Jenna couldn't find them on the shelf.
A. NO CHANGE
B. those
C. it
D. her own
2. Troy walked all the way down the mountain. After that, he was too tired to stay
awake through the movie.
F. NO CHANGE
G. After this,
J. So that
Answers: 1. D, 2. G, 3. C, 4. H
Subject-Verb Agreement for ACT English: Tips
and Practice
In this article, we shall delve into the fun-filled world of subject-verb agreement on the
ACT. Subject-verb agreement questions on ACT English are less common than
punctuation questions; however, you can count on having at least a couple of subject-
verb agreement on your ACT English section, so understanding this grammatical rule
can easily help you improve your ACT English score.
While the grammar rule itself is relatively simple, the questions related to it can be
challenging and a bit tricky. In this article, I’ll teach you strategies and tips to become
a master of all things subject-verb agreement on the ACT.
What is the subject? Justin. Why? Well, he is the hero who did the rescuing.
Similarly, in a sentence with a description, typically using a form of the verb "to be," the
subject is the noun that is being described. See:
Once again, "Justin" is the subject because he is the person who is not funny.
Now that we understand the concept of a subject, I'll define and explain subject-verb
agreeement for you.
On the ACT, most subject-verb agreement questions deal with verb forms in the third-
person singular (he/she/it/one) and third-person plural (they). In the present and present
perfect verb tenses, third-person singular verb forms end in an "s." Third-person plural
verb forms do not. Look at this incorrectly written sentence in the present tense:
While you may have intuitively known how to correct this sentence, and you could have
relied on what "sounds right," you should understand why the original sentence was
incorrect. The subject of the sentence is "the reality television star." She is the
person who does the reading. Furthermore, since we're referring to one reality star, the
subject is singular. Because our subject is singular, the verb needs to be in the
singular form. In the present tense, "read" is plural. Therefore, we need to change the
verb to the singular form "reads."
Let's look at another example with a plural subject. This is our incorrect sentence:
This rule may seem rather simple; however, of course, the ACT doesn't want to make
life too simple for you. Subject-verb agreement questions on the ACT can be
challenging. The sentences will be more complex than the ones above, and the subject-
verb agreement errors will not be as obvious.
Prepositional Phrases
The most common interrupting phrase that appears on the ACT is the
prepositional phrase. Simply, a prepositional phrase is a phrase that begins with a
preposition. What are prepositions? Prepositions provide additional details about nouns
and often answer the questions "Where?", "When?", or provide descriptive information.
Take a look at some examples of commonly used prepositions:
Changes for the new and improved ACT Writing section is going to be implemented
soon.
First, let's identify the subject. What is going to be implemented? Changes. The phrase
"for the new and improved ACT Writing section" is a prepositional phrase that begins
with the preposition "for." The prepositional phrase describes the changes that will be
implemented. See how the ACT can trick you by putting a singular noun, "section," right
before the verb.
However, because "changes" is plural, the verb should be in the plural form.
Unfortunately, "is" is singular. Here's how the sentence should look:
Changes for the new and improved ACT Writing section are going to be implemented
soon.
Strategy
Cross out the prepositonal phrase and the sentence should still be grammatically
correct. Additionally, using this strategy will enable you to more easily identify the
subject and determine whether there's an error in subject-verb agreement. Always
remember that the subject will never be contained within a prepositional phrase.
Let's employ this strategy with the incorrectly written sentence above:
Changes for the new and improved ACT Writing section is going to be implemented
soon.
Now that the subject is right in front of the verb, the subject-verb agreement error is
much more obvious. The strategy involves three steps:
Use this strategy and you'll be much less likely to miss a subject-verb agreement
question on the ACT. You're welcome.
Let's use this strategy with an example taken from an actual ACT:
First, let's cross out the prepositional phrase. Equipment for both types of kayaks are
similar. We're left with "Equipment are similar." Even if the answer is obvious to you at
this point, let's go through the remaining steps. The subject is "equipment," which is
singular. The verb is "are," which is plural; therefore, there's an error in subject-verb
agreement and the correct answer is G. Based on the context of the passage, the verb
should be in the present tense.
The other types of interrupting phrases on the ACT serve the same function as the
prepositional phrase in subject-verb agreement questions. They separate the subject
from the verb. Let's take a look at another common type of interrupting phrase on the
ACT.
Non-Essential Clauses and Appositives
One additional note about non-essential clauses is that a subject-verb agreement error
can be contained within the clause itself. Take this sentence, for example:
The verb in the non-essential clause, "is," corresponds with the subject "carbonated
beverages." Because the subject is plural, the verb should be in the plural form as well.
The sentence should read:
Carbonated beverages, which are delicious, are not good for your health.
While a non-essential clause usually starts with a relative pronoun (which, who, whose,
or where), it doesn't in a phrase known as an appositive. An appositive serves the
same purpose as a non-essential clause, but an appositive doesn't contain a verb.
Here's a sentence with the appositive in bold:
The appositive provides descriptive information about my sister, but the phrase can be
removed without changing the meaning of the sentence or creating a grammatical
error.
How will knowing about these phrases help you answer subject-verb agreement
questions on the ACT? Read below to find out.
The non-essential clause separates the subject from the verb. The subject is "boss,"
which is singular, and the verb is "give," which is plural. There's an obvious error in
subject-agreement. The sentence should read:
On the ACT, you may encounter a similar sentence. Many students will incorrectly
assume that the subject is "employees" because that is the noun closest to the verb.
While "employees" is plural, the actual subject "boss" is singular. By correctly identifying
the subject, you can avoid being tricked by the interrupting phrase.
The method you should you use to avoid being tricked is the same one I taught you for
prepositional phrases.
Strategy
After we cross out the non-essential clause, we're left with "My boss give helpful
advice." The singular subject is right next to the plural verb. The error in subject-verb
agreement should be obvious. Nice!
Let's put our knowledge to use. Here's another example from a real ACT:
In this example, the non-essential clause is surrounded by dashes, which serve the
same function as commas in this sentence. The ACT folks threw in that extra curve ball
because they also really like testing your knowledge of punctuation. If we cross out the
prepositional phrase "of letters" and the non-essential clause, we're left with
"Dickinson's last twenty years reveals." The subject is "years," which is plural, so
the verb should be in the plural form. The answer is B.
Let's briefly discuss one last type of interrupting phrase you may encounter on the ACT.
A book that has an interesting plot and well-developed characters are fun to read.
The essential clause begins with "that" and ends right before the verb "are." Just like we
did with non-essential clauses, let's cross out the clause. We're left with "A book are
fun to read." The singular subject is now right next to the plural verb. Here's the
corrected version of the sentence:
A book that has an interesting plot and well-developed characters is fun to read.
There's another tactic the ACT uses, though much less common, to complicate basic
subject-verb agreement questions.
In rare instances, the typical word order of a sentence will be altered so that a
prepositional phrase appears at the beginning of a sentence and the subject follows the
verb. In these instances, it can be particularly challenging to identify the subject and
determine if there's an error in subject-verb agreeement. In order to illustrate this point,
let's look at an example sentence with the prepositional phrase underlined and the verb
in bold:
We know that a subject can't be contained within a prepositional phrase. Also, we know
that a subject is the noun that's doing the action. In the sentence, what's residing? The
water bottles. The prepositional phrase provides information about where the water
bottles are residing. Because the subject is "water bottles," the verb should be in
the plural form. This is the correct version of the sentence:
In extremely rare situations, the subject follows the verb but is not preceded by a
prepositional phrase. Check out this sentence:
In this sentence, what is less fun? The "consequences". Therefore, in the second
clause, "consequences" is the subject. The verb "is" corresponds with the subject
"consequences." Because we have a plural subject and a singular verb, there's an
error in subject-verb agreement. This is the corrected version of the sentence:
Is there a way to simplify these types of sentences to help determine if there's an error
in subject-verb agreement? Of course.
Strategy
In sentences in which the subject follows the verb, rearrange the sentence so that it
follows the normal structure of subject then verb. Using this strategy will allow you
to more easily spot any errors in subject-verb agreement.
If you utilize this method in the incorrectly written sentence above, you'd be left with "its
consequences is less fun." The plural subject is now right next to the singular verb
and the error should be immediately apparent.
Now, we'll cover one more unique situation that can complicate the most basic subject-
verb agreement questions.
Because the subject is both "Justin" and the "ACT," the verb should be in the plural
form.
First, let's rearrange the sentence so that the subject comes before the verb. What's in
between the cushions? Change and an old pen. So after doing our rearranging, the
sentence should read "Change and an old pen is in between the cushions of my couch."
The subject is both change and an old pen; therefore, the verb should be in the plural
form. This is the corrected sentence:
At this point, we're aware of the ways the ACT complicates basic subject-verb
agreement questions. We've also learned specific strategies to use when faced with
certain types of situations. Here are some general tips to follow to help you correctly
answer any subject-verb agreement question you may encounter on the ACT.
Singular and Plural Forms of the Same Verb in the Answer Choices
Signal a Possible Error in Subject-Verb Agreement
On the ACT, you can often figure out what's being tested based on the underlined
phrase and the answer choices. If you see that there are singular and plural forms of the
same verb in the answer choices, determine if there's an error in subject-verb
agreement.
Both subjects of sentences and subjects of clauses must agree with their verbs. For
each underlined verb, find the noun that corresponds with that specific verb. Then,
determine whether that subject is singular or plural and make sure that the subject and
verb agree.
Be aware that a subject will not be part of a prepositional phrase. Most subject-verb
agreement questions on the ACT separate a subject from a verb with a prepositional
phrase. Remember the strategy of crossing out the phrase to aid in answering these
questions.
Knowing the common tricks the ACT English section uses on questions that test your
knowledge of subject-verb agreement can be helpful. The better you know these tricks,
the more quickly you'll be able to recognize them and use the appropriate strategies to
correctly answer subject-verb agreement questions.
Additional Practice
Hopefully, by this point you thoroughly understand subject-verb agreement and how to
correctly answer any subject-verb agreement question that may appear on the ACT. I've
created some realistic ACT English practice problems to test you on what you've
learned. Remember to use the general strategies I referenced above.
A. NO CHANGE
B. have been
C. was
D. is being
A. NO CHANGE
B. was
C. will being
D. are
3. The selfish man, who owns five cars and two houses, has been unwilling to give any
money to charity.
A. NO CHANGE
B. were
C. have been
D. has being
A. NO CHANGE
B. are being
C. is
D. were
Answers: 1. B., 2. D, 3. A, 4. C, 5. A
However, the ACT English section isn't writing—it's a multiple choice test, which
means that every question has to have only one correct answer. The ACT tests a
specific set of punctuation rules, most of which deal with commas. I covered commas in
a separate post—here I'll be explaining the other punctuation rules you need to know,
which deal with apostrophes, semicolons, colons, and dashes.
Possessives
If a word is singular or if it's plural but doesn't end in "s," then you add "'s" to the end of
the word.
To create a possessive for a plural word that does end in "s," you simply place an
apostrophe at the end of the word, after the "s."
At the tinkers convention, Jose browsed for a long time before deciding he wanted to
buy something from the leather workers stall.
Clearly, this sentence is missing a number of apostrophes. The convention is for a lot of
tinkers, so there should be an apostrophe after the s. It's probably just one leather
worker at the stall though, so the apostrophe should go before the s.
At the tinkers' convention, Jose browsed for a long time before deciding he wanted to
buy something from the leather worker's stall.
On the ACT English, possessives questions aren't so much about knowing the
rules as they are about determining whether you need an apostrophe at all and, if
so, whether the noun is meant to be plural or singular.
When my computer crashed, I lost a months work on my plan to take over the world.
Should month be possessive? Try switching it around:
When my computer crashed, I lost a month's work on my plan to take over the world.
It's easy to assume that because a month isn't a person it can't be possessive, but that's
not the case. Any noun can be possessive.
In general, when dealing with possessive nouns, you're more likely to see questions
where there's a necessary apostrophe missing (or misplaced, which we'll discuss
shortly), than you are to see ones with extra apostrophes that you don't actually need.
The other factor you'll need to consider when placing apostrophes is whether the
possessive noun is meant to be singular or plural. Whether "the neighbor's dog" or "the
neighbors' dog" is correct is dependent on whether there is only one neighbor or more
than one. When working on ACT English questions, you'll need to look at context
clues to determine how many of the noun there are meant to be.
When my computer crashed, I lost a months work on my plan to take over the world.
We've already established that "months" should be possessive. But how do we know
that it's "month's" not "months'"? The singular article "a" gives us the clue—"a months"
makes no sense, so the noun has to be singular.
ACT Example
The ACT tests possessives in almost exactly the same way we just discussed. Take a
look at this example from an official ACT English section:
Should "family" be possessive? If we switch around the word order to "the farm of his
family" that makes sense, so yes, the noun should be possessive. (In this example, you
might also say "family farm" but you'll notice that isn't an answer choice.)
Is it "family" or "families"? First, think about what clues you can use—"his" is a good
one. We're talking about Banneker's family specifically, and people generally only have
one family. The singular form is correct.
Now we know that we need an answer with "family" and an apostrophe, which narrows
it down to A and B. We need to pick the one that correctly places the apostrophe before
the "s" (since we're dealing with a singular noun), which is B.
Contractions
When working with contractions, you only really need to know one rule: the apostrophe
replaces the missing letter or letters.
There are a few weird cases like "won't," but you don't need to worry about them. In
fact, the ACT English generally doesn't ask about spelling of contractions. Why are we
going over this rule then? Because there are a couple of very common contraction-
related errors that the ACT English does test.
If you've done any ACT English practice you may have seen the construction "could of,"
"should of," or "would of." For example:
I should of gone to bed early, but I stayed up to play video games instead.
This sentence sounds fine if you read it aloud, but it's actually incorrect. I'm trying to say
that I should have gone to bed—the correct spelling is "should've" not "should of":
Pronouns
One of the most common mistakes students make on the ACT English is mixing up
"it's," "its'," and "its." Do you know the difference?
It's — it is or it has
Its — possessive form of it
Its' — not a word
Think about it this way: a contraction must have an apostrophe to replace the dropped
letters while no other pronoun possessives (his, her, my) have apostrophes. Its',
meanwhile, is just a weird construction that only shows up on the ACT—it is never
correct.
Though they're less common, errors with they're, there, and their and errors with who's
and whose may also appear on the test. For more details on those, take a look at our
post on word choice.
We're going to go over the exact uses for each of these punctuation marks, but, first a
quick disclaimer. Semicolons, colons, and dashes are almost always tested with
commas or in the context of sentence fragments and sentence structure. We have
separate posts on each of those, so make sure to look at them as well.
Semicolons
Semicolons are basically wishy-washy periods (or so their detractors claim); they
connect two independent clauses. That previous sentence is an example! A semicolon
is only correct if it could be replaced with a period.
Incorrect: After seeing that the amusement park had four roller coasters; Maria was
determined to ride them all.
Correct: The amusement park has four roller coasters; Maria was determined to ride
them all.
In the first version of the sentence, a semicolon is being used incorrectly in place of a
comma to connect a dependent and an independent clause. In the second version, it is
correctly placed between two independent clauses.
Remember that semicolons are interchangeable with periods—this means that you will
never be asked to choose between them. If the only difference between two answers
is that one has a period and the other semicolon in the same spot, both answers
must be wrong.
There is one other, much less common, use of semicolons: to separate items in a list,
especially if they include commas. For example:
My whole family was at the reunion, including my cousins, Kirsten and Jeremy; my
aunts, Tracy and Megan; and my grandparents, Carl and Jen.
Again, there's a small chance you'll see this usage on the test, but it's very small. Don't
worry about it too much.
Colons
Colons are easy to mix up with semicolons because the two punctuation marks look
similar and have similar names. Colons can, in fact, connect two independent clauses,
but they are usually used to introduce lists or explanations (you may have noticed
that I tend to employ them for the latter purpose quite a lot).
The key rule for colons is that they must come after a complete sentence. You should
be able to put a period at the end of the clause before the colon and have it make
sense. Otherwise, it doesn't matter if you're introducing a list or explanation, it's still
wrong.
Incorrect: Liz went to the costume shop for the supplies she needed for Halloween,
including: fake blood, plastic spiders, and a witch’s hat.
Correct: Liz went to the costume shop for the supplies she needed for Halloween: fake
blood, plastic spiders, and a witch’s hat.
Correct: Liz went to the costume shop for the supplies she needed for Halloween: a
holiday when people dress up in scary costumes and eat a lot of candy.
Correct: Liz went to the costume shop for the supplies she needed for Halloween: she
was planning to dress up as a zombie witch.
The first sentence is incorrect, because the part that comes before the colon isn't a
complete thought; "Liz went to the costume shop for the supplies she needed for
Halloween, including" makes no sense on its own. The other three versions all correctly
locate a colon at the end of the independent clause, "Liz went to the costume shop for
the supplies she needed for Halloween."
You may have noticed that semicolons, colons, and periods can all be used
between independent clauses. However, as I mentioned in regard to semicolons, you
will never be asked to choose between the three—the distinctions are purely
stylistic. If you have more questions on this topic, read our guide to sentence fragments
and run-ons.
Dashes
Non-essential clauses and phrases provide extra information that can be removed
without altering the meaning of the sentence. When dashes are used with non-essential
clauses or phrases, the key is making sure that you don't mix them with commas. Both
are equally correct, but you have to stick to one or the other.
Incorrect: Allie was minding her own business when her brother, a mischievous eight-
year-old—snuck up and surprised her.
Correct: Allie was minding her own business when her brother—a mischievous eight-
year-old—snuck up and surprised her.
Correct: Allie was minding her own business when her brother, a mischievous eight-
year-old, snuck up and surprised her.
To mark off non-essential clauses is by far the most likely way you'll see dashes tested
on the ACT English. However, you may also see dashes used to introduce a list or
explanation, like so:
Allie’s eight-year-old brother surprised her—he snuck up behind her and yelled “boo!”
Dashes are relatively rare on the ACT, so just understand roughly how they can be
used and you'll be fine.
ACT Example
As I mentioned above, these punctuation marks are often tested together. You can see
what that looks like in this official ACT question:
On questions like this one, it's often easier to determine what doesn't work than what
does. So let's start with the semicolon; "but versatile boat" definitely isn't an
independent clause, so we know a semicolon can't be correct. This construction
also isn't a non-essential phrase (if you take it out the sentence no longer makes
sense), a list, or an explanation, so we can rule out the dash.
The comma is tempting because it's right before the coordinating conjunction "but," but
it's actually incorrect. Think about what commas, dashes, colons, and semicolons are all
meant to do: clarify relationships between clauses and phrases. That means you
generally shouldn't stick them in the middle of a thought. "simple but versatile boat" is
one idea, so you don't want to interrupt it with unneccesary punctuation. B is the correct
answer.
Possessives: the apostrophe goes before the s for singular nouns and plural
nouns that don't end in s and after the s for plural nouns that do end in s
Contractions: the apostrophe replaces the missing letters
Possessive pronouns don't have apostrophes
Semicolons connect two complete sentences
Colons come after a complete sentence and introduce a list or explanation
Dashes mark off non-essential clauses or introduce lists and explanations
Helpful ACT English Strategies:
1. The two main types of camels are; bactrian camels, which have two humps, and
dromedary camels, which have one.
A. NO CHANGE
B. are: bactrian
C. are bactrian
D. are, bactrian
F. NO CHANGE
G. lollipops'
H. lollipop's
J. lollipop
A. NO CHANGE
B. Joe,
C. Joe who
D. Joe —
4. Even though she's been trying to cut it down, Fran's action album collection is
still impressive: over 400 vinyl records, 2000 CDs, and even a handful of tapes.
F. NO CHANGE
G. impressive; over
H. impressive over
J. impressive, it's
Answers: 1. C, 2. G, 3. D, 4. F
You may recall the good ol’ days of elementary school when you learned
about adjectives and adverbs. If you had realized that you would have to know about
these parts of speech for the ACT, perhaps you would have paid better attention instead
of daydreaming about Nickelodeon and The Disney Channel. Don’t worry. If you’ve
forgotten or never learned about these parts of speech, I’ll teach you everything you
need to know about them for the ACT English section.
Define an adjective.
Define an adverb.
Review the comparative and superlative forms of adjectives.
Explain how adjectives and adverbs are tested in ACT English.
Provide practice questions to test you on what you've learned.
What Is an Adjective?
Adjectives modify nouns or pronouns. Here's an example:
The word "boring" is the adjective because it modifies the noun "movie." Check out one
more example sentence with an adjective:
The word "diligent" modifies the noun "student." Adjectives describe or provide more
information about a noun.
What is an Adverb?
In elementary school, you may have learned that adverbs modify verbs, but that's not
all. Adverbs modify verbs, adjectives, or other adverbs. This is an example of an adverb
modifying a verb:
The word "incredibly" modifies the adjective "boring" that modifies the noun "passage."
And, finally, this is a sentence with an adverb modifying another adverb:
The word "extremely" modifies the adverb "slowly." The word "slowly" modifies the verb
"walk."
You may have noticed the adverb form is typically created the same way.
Adverb Construction
Adverbs are usually formed by adding "ly" to the adjective. For adverbs that end in
"y," the adverb is formed by adding "ily." Here are some examples: "quick" becomes
"quickly," "soft" becomes "softly," "close" becomes "closely," and "hasty" becomes
"hastily." So if you say that somebody "talks slow" or "drives careful," you're making a
grammar error. You should say, "talks slowly" or "drives carefully."
A. NO CHANGE
B. amazing poorly
C. amazingly poor
D. amazing poor
In the sentence, "amazingly" modifies "poorly." The word "poorly" modifies the singer.
Because "singer" is a noun, "poorly" should be in the adjective form. Only adjectives can
modify nouns. Because "amazingly" modifies an adjective, it should remain in the
adverb verb. The answer is C.
Some sentences will use an adjective in the place of an adverb or vice versa:
It was the sun that was powerful, not "summer." Because only an adjective can
modify a noun, "powerfully" should be in the adjective form. This is the correct version of
the sentence:
How should you go about figuring out adjective vs. adverb questions on the ACT?
Strategy
Determine what word an adjective or adverb is modifying. Then, determine the part of
speech of the word that is being modified to check to see if the adjective or adverb is
being used correctly. Adjectives can only modify nouns and adverbs can only modify
verbs, adjectives, or other adverbs. Make sure that adjectives are in the adjective form
and adverbs are in the adverb form.
Apply these tips to an adjective vs. adverb question from an actual ACT.
On each wing, all flighted birds have ten primary flight feathers, each one
shaped slight different.
F. NO CHANGE
G. slight differently.
H. slightly differently
J. slightly more different
Explanation: First, we see that both underlined words are in the adjective form. Next,
we have to determine the function of each word in the sentence. Is each word being
properly used as an adjective? Let’s start with “different." What is “different” modifying?
How the feathers were shaped. “Shaped” is a verb. Therefore, “different” should be in
the adverb form, “differently." What is “slight” modifying? The adverb “differently."
Therefore, “slight” should also be in the adverb form. The answer is H, “slightly
differently."
Now, let's take a look at another issue tested on the ACT that involves adjectives and
adverbs.
Comparatives
Typically, you use the "er" form for words with one syllable and "more" + adjective for
words with two or more syllables. One exception is that two syllable words that end in
"y" tend to use the "er" form. Examples: "funny" becomes "funnier" and "busy" becomes
"busier." Here's an example of a comparative being used correctly:
The comparative "easier" is being used to compare two things: Macs and PCs. So what
form do you use to compare three or more things?
Superlatives
The superlative form of an adjective is formed by adding "est" to the word or "MOST"
+ the adjective. Examples of superlatives include "strongest," "lightest," and "most
fascinating." You can never use "most" with the "est" form. It's incorrect to write "most
funniest" or "most strongest." The superlative form is only used when comparing
three or more things. Typically, words with one syllable use the "est" form and words
with two or more syllables use "most" + adjective. This is an example of a correctly used
superlative:
When using "all" and not specifying a number, it's implied that you're talking about more
than two so you should use the superlative form. Check out this example of a
superlative being used incorrectly:
Remember that the superlative can only be used when comparing three or more things.
There are only two in this sentence: John and Suzy. To fix the error, "quietest" should
be changed to the comparative form, "quieter."
How does the ACT test your knowledge of comparatives and superlatives?
Here are some tips to help you solve comparative and superlative questions on the
ACT.
Strategy
Comparative and superlative rules are relatively basic. Remember this information and
you should be able to correctly answer any ACT English question about them.
If a comparative is underlined, make sure only two things are being compared. Never
use "more" with the "er" form. Use the "er" form for one syllable words. If a superlative is
underlined, make sure three or more things are being compared. Never use "most" with
the "est" form. Use the "est" form for one syllable words.
Use your knowledge of comparatives and superlatives to answer this real ACT English
question.
The two principal types of kayaks are the easily maneuverable white-water kayak
and the largest sea kayak.
F. NO CHANGE
G. very biggest
H. more large
J. larger
Explanation: Because the word underlined is in the superlative form, we need to check
to see if the superlative form is being used correctly. What is being compared in the
sentence? The white-water kayak and the sea kayak.
Only two things are being compared, so you have to use the comparative form. The
sentence even says, “The TWO principal types of kayaks." Also, “large” is a one syllable
word so you should use the “er” form instead of “more large." The answer is J.
Here are some tips to help you answer all questions relating to adjectives and adverbs
on the ACT.
#2: Determine the Part of Speech of the Word that the Adjective or
Adverb Is Modifying
Comparatives
Comparatives only compare two things. Proper comparative structure is the "er" form or
"MORE" + adjective. Never use "MORE" with the "er" form. Use the "er" form for one
syllable words.
Superlatives
Superlatives compare three or more things. Proper superlative construction is the "est"
form or "MOST" + adjective. Never use "MOST" with the "est" form. Use "est" for one
syllable words.
Additional Practice
I created these realistic practice problems to test your knowledge of adjectives and
adverbs on the ACT. Enjoy!
1. The young student was disillusioned with school; he found his classes boring, and he
thought economics was his most boringly class.
A. NO CHANGE
B. most boring
C. more boring
D. more boringly
3. Chris Farley was a very gifted entertainer who left an extremely profound impact on
Saturday Night Live.
A. NO CHANGE
B. extremely profoundly
C. extreme profound
D. extreme profoundly
A. NO CHANGE
B. more prestigiously
C. most prestigious
D. most prestigiously
Answers: 1. B, 2. A, 3. A, 4. C
What's Next??
Keep improving the skills that will help y
Hence the Writing SAT was born! While there are obviously some issues with this
section of the test - such as the essay - which are being addressed in the new version
of the test, there is a consensus that the ability to create strong paragraphs is a more
important skill than being able to recognize tricky grammar errors in isolated sentences.
In fact, the new version of the SAT that will be introduced in 2016 will place an
even greater emphasis on passage-based, grammar-in-context questions.
The test covers a few main editing concepts: how to correctly use transitions, good
organization, conciseness, eliminating and adding sentences, meaning and
purpose, how to improve sentences, and specificity. Later, I will give you specific
strategies for attacking each of these question types. If you want to learn more about
these different kinds of questions, see our guide to the concepts tested on the SAT
Writing.
But first...
How should I approach Improving Paragraphs
questions?
When you hit this section of the test, follow these rules for the correct approach.
Remember that while some of the questions look similar to Improving Sentences or
Critical Reading questions, they differ in some important ways!
You will usually notice that the passage is not very well written: it is meant to look like
the first draft of a hastily-written essay.
You do NOT need to read the passage carefully at this point. Unlike in the Critical
Reading portion of the test, you are not being asked to understand the purpose or
deeper meaning of the passage. You only need to be able to notice mistakes in
logic and grammar.
2. How is the piece organized? How do the paragraphs relate to each other?
3. Was anything confusing - either because it could have been explained more clearly,
or because the way it was worded made it difficult to understand? If so, mark it.
4. Did anything seem glaringly out of place or unnecessary? If so, mark it.
While these seem like basic questions, the majority of the questions you will be
asked will revolve around these concepts.
If the question refers to a specific sentence in the passage, go back and read that
sentence more closely, along with the sentences before and after it. This is because
this section of the test focuses on context.
Without understanding the context, it will be much more difficult to weed out incorrect
answers. Any answer you choose has to not just be grammatically correct, but also has
to fit in with the sentences that surround it.
Step Five: Make sure that your answers are both CONCISE and
GRAMMATICALLY CORRECT
Not a fragment
Not a dependent clause
No unnecessary prepositions or articles
Antecedents must have clear pronouns
Use fewer gerunds and participles if possible
No dangling or misplaced modifiers
Remember, if there are several grammatically-correct options that work equally well in
context, choose the most concise option.
The SAT is testing two things with these questions: can you identify and correct
grammatical errors, and can you understand how sentences work in the context of the
paragraph?
If the question asks about improving a certain sentence or combining two
sentences "in context", make sure that you go back and read the sentence in
question as well as the sentences around it. Your answer should make a concise and
direct sentence that is both correct indiviudally, and also works in the paragraph.
If the question asks about the best way to improve a sentence or combine two
sentences, but does not say "in context", this is going to be very similar to the
Improving Sentences questions in the writing section. Pay particular attention to your
grammar rules, including dangling modifiers, antecedents of pronouns, and subject-verb
agreement. As ever, also look out for the most concise and direct way of saying the
information.
2. Transitions
Improving paragraphs questions will often test your ability to add transitions. Sometimes
these will be single-word or short phrase transitions (do you know when to use
"furthermore" as opposed to "therefore"?). Other times you will be dealing with full-
sentence transitions.
With these kinds of questions, the SAT is looking to see if you know how to lead
smoothly from one idea in the paragraph to the next.
When asked to add a transition word or phrase, go back and look at the sentence in
question and the previous sentence. Try to understand the relationship between the two
setnences. Does the second explain the first? Does in contradict the previous
sentence? Does the sentence in question reach a conclusion?
The table below reviews some common transition words and what they are used for:
At other times, you will be asked to add a whole sentence between two other
sentences or at the start of a paragraph. In these cases, you are looking for full
transition sentences. Read the sentences that will surround the new sentence and
chooise the option that best relates the idea in the first sentence to the idea in the
second sentence.
3. Organization
Some questions will ask where a sentence best fits within a paragraph. This is all about
logic. If a sentence is breaking up two other sentences about the same topic that should
be together, then it needs to be moved. If the sentence is giving more information about
a topic that has not been introduced yet, it needs to be moved.
4. Conciseness
Frequently, the SAT will ask you the best way to combine two sentences. This means
that as they are, the two sentences are too wordy. Is there a way that these sentences
could be joined together that is more effective?
Other times you may be asked the best way to re-write an overly-wordy sentence. If the
sentence has extra words that are adding no benefit, then you can get rid of them.
Think: what is the most direct way to say what the author is trying to convey?
Can you understand why the author wrote something in a certain way? If the author has
written something non-serious, do you understand that it is meant to be humorous? Do
you recognize that a certain sentence is giving an example, or introducing a new
argument?
If the question asks about the author's goal in writing a sentence or group of
sentences, go back an re-read the section carefully.
Choose the answer that you can relate directly back to the passage in question.
These questions are a bit similar to Critical Reading questions - remember that there
will ever only be one correct answer and you need to be able to find it in the text!
Be careful when looking over the answer choices and remember that if even one word
in the answer choice doesn't match the section, then you have to rule out that answer.
The SAT will often ask if a sentence should be eliminated. If the sentence is about a
new concept that otherwise is not discussed in the essay, can you tell that this
information is irrelevant?
On the flip side, do you understand that some ideas should be fleshed out with an
example or two? Can you recognize where it's best to add an extra sentence to
describe something previously discussed?
If it asks you to add a sentence, choose the one that best builds on something
that has already been discussed. Try to find a sentence that gives a good example of
whatever the previous sentence had to say. Never introduce a new topic or subject
matter in an added sentence.
If it asks you if a sentence should be deleted, see how well it relates to the
information around it. The SAT often thinks that sentences should be deleted if they
are too different from what else is around them, or if they discuss information that is not
discussed elsewhere in the essay. If it's not relevant, get rid of it!
7. Specificity
Many of the passages will use vague words that could be interpreted in several different
ways, depending on the context. Can you understand how a word is being used in the
passage?
Try mentally substituting the answer words into the sentence if it is not immediately
clear.
(1) Many people consider reality television vapid and a way to perpetuate bad
stereotypes. (2) The current season of The Bachelorette elicited harsh reviews from feminists,
they are people who believe in the equal representation of men and women. (3) The most
condemning ones claimed that gender relations were being set back by decades because of the
new "twist" of having two bachelorettes. (4) Wouldn't young girls watching the show internalize
the inherent message when having men pick the bachelorette based on looks, before the
bachelorette could pick a spouse based on his personality? (5) And WifeSwap, the television
show in which men trade wives for a week. (6) Imagine the horror at having women portrayed as
a belonging that can be borrowed!
(7) I see nothing wrong with reality television, whether low-brow or of the skill-based
competition variety. (8) After all, isn't the drama of our daily lives the most entertaining thing of
all? (9) For example, my friend's love life has enough ups and downs to be an award-winning
drama. (10) Reality television would never have become so popular if we did not naturally like
watching the struggles of every day life. (11) No doubt, in The Bachelorette we recognize the
desire for connection and love, this is reminiscent of the plots of classic movies such
as Casablanca, directed by the late Michael Curtiz. (12) Curtiz will see traces of his own love
triangles in the contestants on these shows. (13) The bachelorette woos the main contestant,
before finally breaking his heart on the final episode and leaving him for someone else.
(14) Human drama and competition have been the main compelling forces in all entertainment
for centuries, and we should not forsake them in favor of something more predictable but safer.
1. Which of the following is the best version of the underlined portion of sentence 11
(reproduced below)?
No doubt, in The Bachelorette we recognize the desire for connection and love, this
is reminiscent of the plots of classic movies such as Casablanca, directed by the late Michael
Curtiz.
(A) (as it is now)
(B) love, this would be
(C) love; they are
(D) love for being
(E) love as
You may notice that this is not too different from an Improving Sentences question.
Context does not matter here - instead, we just need to fix this to make it as
grammatically and stylistically correct as possible. As it is now, we have a comma
splice, which needs to be fixed. Therefore, (A) is incorrect. (B) also present a comma
splice and so is also incorrect. (C) technically makes a grammatically correct sentence
because the independent clauses are separated by a semicolon. However, the pronoun
"they" is incorrect and does not work in context. The original sentence uses a singular
pronoun referring back to "desire", but "they" is a plural pronoun so cannot refer to
desire. (D) is awkward and uses a gerund where it is not necessary. (E), therefore, is
the correct answer. Of the grammatically correct choices, it is the most concise.
2. In context, which of the following is the best word to use instead of "ones" in sentence 3?
(A) instances
(B) shows
(C) reviews
(D) sections
(E) characters
Your key words here are “in context”, which means that you need to not only read
sentence 3, but also the sentences around it. Sentences 2-3 say, “The current season
of The Bachelorette elicited harsh reviews from feminists, they are people who believe
in the equal representation of men and women. The most condemning ones claimed
that gender relations were being set back by decades by the new "twist" of having two
bachelorettes.” Here it is clear that “ones” is referring back to the reviews, some of
which were particularly critical. Therefore, (C) is the correct answer.
If you were uncertain about this, you could easily sub in the other options and you would
see that they would not make sense in the context of sentence 3.
3. An important strategy used in the first paragraph is to
(A) elaborate on a view that contrasts with the essay's argument
(B) use descriptive detail to elucidate a personal experience
(C) provide a thoughtful, objective analysis of feminism
(D) introduce an unconventional response to reality television
(E) reveal the sense of playfulness implicit in feminist theory
From skimming the full essay, you should see that the author starts by saying that some
people do not like reality television, but the author of the essay thinks there is nothing
wrong with it. Therefore, the correct answer is (A). The first paragraph goes into detail
about why people do not like reality TV, which is a contrast to the author’s point of view
that is argued in the rest of the piece.
What if you weren’t sure? (B) is incorrect because no personal experiences are
mentioned in the first paragraph. (C) and (E) are incorrect because there is no
discussion of feminism. (D) is incorrect because the first paragraph tells that the dislike
of reality television is not an unconventional response - it contrast, it is common.
4. In context, which of the following is the best way to revise the underlined portion of sentence
12 (reproduced below)?
Curtiz will see traces of his own love triangles in the contestants on these shows.
(A) Curtiz could have seen
(B) Curtiz, too, would see
(C) However, Curtiz might have seen
(D) In addition to this, Curtiz will see
(E) Likewise, Curtiz can see
Key words, once again, are “in context.” So let’s go see the context: “No doubt, in The
Bachelorette we recognize the desire for connection and love, this is reminiscent of the
plots of classic movies such as Casablanca, directed by the late Michael Curtiz. Curtiz
will see traces of his own love triangles in the contestants on these shows.
The context shows us that we need a way to link the recognition that we have (of the
desire) with Curtiz's would-be recognition of love triangles. This calls for a transition.
Option (A) does not give a transition at all, so it's out. (C) uses a transition ("however")
but this is not the correct one - this implies a contradictory idea, and we need something
that shows agreement. So (C) is out. (D) and (E) are both out because of the tense of
the verbs. Though they use good transitions that show agreement, Curtiz is dead and
will not be doing anything in the present or future tenses.
That leaves us with (B), which uses an appropriate transition, and also uses a form of
the verb that implies a hypothetical situation. It is also direct and concise - note that it
is the shortest of all the options.
(C) An explanation of why seeing "real-life struggles" on television resonates with viewers
This question is asking what we should add to the first paragraph, which means that we
need to go back and read it. Remember, we should not add information about anything
that is not directly relevant.
Let's start by reviewing the purpose of the first paragraph. In this paragraph, the author
writes about why certain people object to reality TV, and tells what some of those
objections are, using The Bachelorette as an example.
Therefore, a quote about why someone loves reality TV would not fit well in this
paragraph. (B) is out. Paragraph two discusses watching real-life struggles on
television, and also mentions Michael Curtiz's. However, neither of these are mentioned
in the first paragraph, so (C) and (E) are out.
Other types of television programming would be completely off-topic, because this
passage is specifically about reality television. Therefore (D) is out.
In contrast, a reader who is unfamiliar with The Bachelorette may be helped with an
explanation of how the show works, since the show is discussed for much of the first
paragraph. (A) is the correct answer.
Brace yourself for some more ACT English fun. Add/Delete questions are a common
type of rhetorical skills question on the ACT English section.
Add Questions
At this point in the essay, the author is considering inserting the following sentence:
Should this sentence be inserted?
Add questions are two part questions. First, you have to determine if the sentence
should be added and if it logically fits at that point in the passage. Then, you have
to identify the reasoning for why the sentence should or shouldn't be added.
Delete Questions
A. Kept, because...
B. Kept, because...
C. Deleted, because...
D. Deleted, because...
These are also two part questions. You have to determine if a sentence or phrase is
relevant at that point in the passage. If it's not relevant, then it should be deleted; it
has to be necessary in the context of the passage.
Type #2: Primarily Lose
The writer is considering deleting the phrase or sentence...If the phrase or sentence
was deleted, the sentence or essay would primarily lose:
For these questions, you have to analyze the purpose of a phrase in a sentence or the
purpose of a sentence in a passage to determine what the sentence or phrase provides
to the passage.
Strategies for Add/Delete Questions
After you identify an add or delete question, you need to know the strategies and steps
for correctly answering the question. First we'll go over how to tackle add questions.
Add Questions
The following excerpt is taken from the last paragraph of a passage about fixing raptor
feathers:
75. At this point, the writer is considering adding the following true statement:
The imping procedure is just one of the many responsibilities bird rehabilitators have.
A. Yes, because it reveals the relative importance of imping compared to the other
work of bird rehabilitators.
B. Yes, because it reinforces the idea that imping is of great benefit to raptors.
C. No, because it goes beyond the scope of the essay, which focuses on how the
feathers of certain types of birds are repaired.
D. No, because it undermines the essay's earlier claim that imping is the most
important work that bird rehabilitators do.
#1: Analyze the Added Sentence to See What It's Doing
The excerpt that I gave you describes the imping procedure. Earlier in the passage, the
definition of imping is given:
The added sentence is discussing the activities of bird rehabilitators more generally. It's
placing the focus on bird rehabilitators and what they do.
#2: Refer Back to the Passage to See if the Sentence Should Be Added Given
Where It Would Be Placed
The added sentence must be relevant in context. For our example, the sentence should
be related to imping.
For ACT English passages, you can typically determine main ideas with the title, and, if
necessary, the first couple of sentences of the passage.
Raptors, or birds of prey, cannot afford to be grounded for weeks waiting for a large
number of flight feathers to regrow. They must be able to fly if they are to hunt and eat.
Based on just the title and the first couple of sentences, we can determine that the focus
of the passage is on fixing raptor feathers.
Therefore, the sentence would not be an appropriate conclusion because the passage
is not focusing on bird rehabilitators or their responsibilities but on the importance and
process of fixing raptor feathers.
Because the sentence would not logically fit in the context of the passage, we can
determine that the answer is no.
#4: Eliminate the Two "Yes" Answer Choices or the Two "No" Answer Choices
Since we have determined that the sentence should not be added, we can eliminate
answer choices A and B.
In my own words, I would say that the sentence should not be added because the
passage focuses on fixing raptor feathers, not on the responsibilities of bird
rehabilitators.
#6: Pick the Answer Choice that Most Closely Matches Your Reasoning
Answer choice D is wrong because nowhere in the passage does the essay claim that
imping is the most important job of bird rehabilitators. Even though it's possible that
imping is the most important job of bird rehabilitators, you can't assume that's true
unless it's directly stated or inferred from what's written in the passage.
Answer choice C is the one that most closely matches our reasoning. The added
sentence would be outside the scope of the essay because the essay's focus is on
fixing raptor feathers.
Delete Questions
Now that we've covered add questions, let's go through how to answer the two types of
delete questions.
The following excerpt is from the first paragraph of a passage on a potter's kiln:
You have to figure out why the phrase or sentence is included at that point in the
passage. What's it doing there?
In our example question, the phrase "and transform glazes to glorious colors" provides
more information about what a kiln does.
#2: Refer Back to the Passage to Determine if the Deleted Portion Is Necessary
Given Its Context and the Passage's Focus
For our example, we have to determine if providing more detail about the purpose of a
kiln is relevant to the essay's focus. Once again, let's look at the title to determine the
main idea. The title of this passage is The Potter's Kiln. Providing more information
about the purpose of a kiln is indeed relevant to the essay's focus.
Because the phrase provides a relevant detail about the passage's focus, the phrase
should be kept.
We can eliminate choices C and D because we have determined that the phrase should
be kept.
I would say that the phrase should be kept because it provides descriptive information
about kilns, the topic of the passage.
#6: Select the Answer Choice that Most Closely Matches Your Reasoning
Answer choice A is wrong because the phrase has nothing to do with the time-
consuming process of painting pottery. The phrase simply mentions that a kiln can
transform glazes to glorious colors. Answer choice B is closest to our reasoning.
If you follow these steps and comprehend the meaning and function of the deleted
portion, you should be able to arrive at the correct answer relatively quickly.
Now, here is the process for answering the second type of delete question.
To determine the purpose of the deleted portion, first look at the sentence without the
deleted phrases. The sentence would read, "By contrast American society has often
been described as one that values youth over age."
In that sentence, the contrast is different and less descriptive. Therefore, the phrases
make the contrast more descriptive and specific.
We can say that the sentence would be less descriptive and have a different contrast if
the phrases were deleted.
The correct answer must state the purpose of the phrases. Incorrect answers can sound
logical, but they will not match the function of the phrases in the specific sentence. Only
consider the specific portion that would be deleted.
Answer choice "F" is wrong. The phrases "the vibrant energy of" and "the wisdom and
experience gained with" do not show a personal or reflective tone. That's not their
purpose in the sentence and there's nothing about those specific phrases that indicates
a personal or reflective tone.
Answer choice G is wrong. I think we can all admit that there is nothing inherently funny
about those phrases.
Answer choice J is wrong. Nowhere in the sentence does the author indicate a
preference. The phrases are both positive.
#4: Select the Answer that Most Closely Matches Your Own
Answer choice H is correct. The phrases are details that illustrate the contrast.
American society does not just value youth over age, but American society values the
vibrant energy of youth over the wisdom and experience gained with age.
Let's review some general tips to help you with all add/delete questions.
In order to answer any add/delete question, you need to figure out what purpose
the addition or deletion serves in the sentence.
The "primarily lose" questions are directly asking you to determine the purpose of a
phrase or sentence.
#2: Refer Back to the Passage to Help Determine the Context of Added
or Deleted Portions and if Added or Deleted Portions Logically Fit
Any added sentence should logically follow the previous sentence and connect to the
following sentence. It should also be relevant to the focus of the paragraph or
passage.
A phrase should be deleted if it's not relevant to the purpose of a sentence. A sentence
should be deleted if it falls outside the scope of the essay or doesn't logically follow the
previous sentence.
#3: For Two Part Questions, Answer the First Part First
For two part questions, you can immediately eliminate the two answer choices that don't
match your answer to the first part of the question.
For "primarily lose" questions, eliminate the choices that don't match the purpose of the
phrase or sentence.
For all add/delete questions, state why something should be added or deleted
or determine what would be lost in your own words, before looking at the answer
choices.
#6: Select the Answer Choice That Most Closely Matches Your
Reasoning
Ideally, you will see an answer choice that is almost identical to your reasoning.
That should be your right answer.
If there's no answer choice that matches your reasoning, work backwards to identify
the correct answer. Start with a possible answer choice and determine if it correctly
answers the question.
Or, if time permits, review the question again to make sure you didn't make a mistake.
“Parts of speech” is a fancy way of saying that every word in the English language can
be put into one of eight categories, depending on how it is used. Of these eight parts of
speech, only 7 are tested on the Writing SAT and on ACT English.
Are you a bit unsure about what, exactly, a preposition is, or can’t tell the difference
between an adjective and adverb? Feeling a bit rusty after learning this years ago? This
article will provide the foundational information you need before moving on to
more complex grammatical concepts. I'll also go over a few important SAT/ACT
grammar rules.
Many of the concepts covered here are things you'll already know if you've studied
grammar in school. Even if you haven't, many (but not necessarily all) of them will seem
natural to native English speakers.
However, if you haven't studied grammar extensively, you can use this as a reference to
help understand the basic ideas that our other guides will not cover. Check back here if
you come across some terms you're unfamiliar with, or if you need to remind yourself of
what something means.
Many of the concepts in this guide are not directly tested on the ACT and
SAT. Instead, these concepts are building blocks that are important for understanding
the why? behind the concepts that are tested. Therefore, do not worry about
memorizing the names of the grammar terms in this guide, just use the concepts.
Once you understand the different rules for each Part of Speech, you will see that these
are relatively straightforward concepts, and they can be used to help you understand
more difficult concepts.
Nouns
If you are not sure if something is a noun or not, try putting "a" or "the" in front of it and
see if it sounds correct. If it does, then it's probably a noun.
Examples
"A cat" and "the cat" sound correct. Therefore, "cat" is a noun.
"A him" and "the him" sound incorrect. Therefore, "him" is not a noun.
Categories of Nouns
Examples
Proper Nouns: Mary, San Francisco, Golden Gate Bridge, University of Notre Dame,
PrepScholar
Concrete Nouns are people, places, or things that you can physically touch.
Abstract Nouns are thoughts, subjects, games, or ideals. These are things, but they
cannot be touched.
Love is an abstract noun.
Examples
Uses of Nouns
Nouns have several possible functions in a sentence. They can be used as the subject,
predicate nominative, appositive, direct object, indirect object, or object of the
preposition. There are other uses for nouns, but we won't go into them now.
Note: Remember that it is not important that you know the names of these functions.
But it is important to understand how nouns can be used, so you can recognize when
one is being used incorrectly.
Subject
The subject of the sentence is the person or thing that is doing the action of the verb.
Example
Mary is the subject of the sentence because she is doing the action.
Predicate Nominative
A predicate nominative comes after a linking verb (see the verb section for more info)
and re-states the subject of the sentence.
Example
Mary is a great friend.
Mary is the subject. Friend is the predicate nominative. In this sentence, Mary and
friend are the same thing, or Mary = friend.
Appositive
An appositive is a noun that re-states or gives more information about another noun in a
sentence. Unlike a predicate nominative, it does not come after a linking verb. Instead,
it's usually right next to the noun it's describing, and is set off by commas.
Because appositives are set off by commas from the rest of the sentence, it usually
works to remove the appositive and still have a grammatically correct sentence.
Example
Here, Mary is an appositive because it gives more information about who the friend is.
Direct Object
A Direct Object is a noun that receives the action of a transitive verb (more on these
here)
Example
Ask yourself: I got what? Got a score. Therefore, score is the direct object.
Indirect Object
Example
I made Kim some brownies.
Ask yourself: I made what? Brownies. Therefore, brownies is the direct object.
Object of a Preposition
All prepositional phrases consist of at least a preposition (see below) and a noun. The
noun that comes after a preposition is called the object of the preposition.
Example
This is another concept that is not directly tested on the SAT and ACT, but occasionally
you will need to recognize if a noun is singular or plural in order to match it with the
correct verb.
Examples
Add -es if the noun ends in -s, -x, -z, -sh, or -ch. This is simply because of
pronunciation - it's really difficult to make these sounds followed by "-s"
Examples
Examples
pony → ponies
fly → flies
Some nouns that end in -f or -fe form the plural by dropping the -f/-fe and
adding -ves.
Once again, this is all about ease of pronunciation, as it's really difficult to make
the -fs sound at the end of a word.
Examples
leaf → leaves
life → lives
Some nouns form the plural irregularly. Unfortunately, there are no rules for how
to form these, and you just have to memorize these words. Fortunately, most of
these words are quite common and you should know most of them already.
Examples
mouse → mice
child → children
man → men
goose → geese
foot → feet
Examples
deer → deer
fish → fish
sheep → sheep
NOTE: NEVER FORM THE PLURAL BY ADDING -'S (apostrope +s). This
should only be used for showing possession.
Examples
Bob's, mouse's, donkey's
To form the possessive of a plural noun that ends in -s, just add an apostrophe.
Examples
To form the possesive of a plural noun that is irregular and does NOT end in -s, add -'s.
Examples
Are you a grammar pro? If you already knew all of the above about nouns, here are a
few special categories of noun you may have been unaware of!
Gerunds
The -ing form of a verb can be used as a verb as long as it has a helping verb. But did
you know that if it's standing on its own, it's called a gerund and is used as a noun?
Example
Infinitive as a Noun
Similarly, the infinitive form (the "to" form) can be used as a noun.
Example
He likes to run.
Here, "to run" is the direct object of the verb "likes". Therefore, it is being used as a
noun.
Pronouns
Pronouns are words that can replace nouns. Unlike nouns, pronouns have different
cases.
What Is a Case?
This means that the form the pronoun takes can change depending on what purpose it
has in the sentence. Above, we saw that nouns can be used either as a
subject/nominative, or as several different types of objects. Personal pronouns have
one form when they are used as a subject or predicate nominative, and another form
when used in any of the object functions (direct object, indirect object, or object of
preposition).
Personal Pronouns
First Person I We
First Person Me Us
The nominative case forms should ONLY be used when the personal pronoun is being
used as the subject of a clause or as the predicate nominative. (Don't remember
what these are? Check out the Noun section above!)
Examples
Many people would say “Michael and him”, but this is incorrect because “he” is a subject
of the clause and therefore must be in the nominative case. When you are not sure, try
crossing out the other subject. You would never say, “Him is my best friend.”
Example
Examples
He gave her and me great presents.
Non-Personal Pronouns
These include but are not limited to: this, that, both, some, few, many, either, which,
who, and one. These pronouns do not change form between the nominative case and
the objective case.
Some people do not think that these pronouns look correct standing on their own, but
they are.
Examples
These pronouns can be tricky to use with verbs because it is not very obvious whether
they are singular or plural. A good way to figure this out is to try to add the word “one”
after the pronoun. Does it make sense? If so, use a singular verb.
Examples
This (one) is my favorite. CORRECT
If adding “one” does not work, mentally add “of them”. If this makes sense, use the
plural verb.
Examples
Exception: The pronoun "one" is always singular, even though "one of them" sounds
correct. Always use a singular verb.
Singular Plural
First Person mine ours
Because these are pronouns (not adjectives), they can be used independently.
Examples
Theirs beat out ours.
The SAT and ACT both frequently test pronouns. Here is a brief summary of what you
are most likely to see tested. For more detail, see our article on pronouns on the ACT
English and SAT Writing.
Rule 1
Always make sure a pronoun has a clear antecedent. The antecedent is the noun that
the pronoun is replacing. If there is not an obvious antecedent for the pronoun either in
the same sentence, or a sentence very nearby, it is considered an error.
Example
This would be considered incorrect because it is not clear who "she" is referring to.
Sometimes the antecedent will be in a previous sentence. In this case, it should also be
very clear.
Example
John Wallach, an award-winning journalist who covered conflicts in the Middle East for
two decades, founded Seeds of Peace in 1993. He chose the campsite because it is a
beautiful natural setting far from the places of conflict. It remains unaffiliated with any
nation, organization, or peace group, and although countries may select their campers,
they cannot pay for the camp.
Here it appears that “it” refers to “campsite” in the previous sentence - really it should
refer to “Seeds of Peace” in the first sentence. This would be considered an unclear
and incorrect use of a pronoun.
Rule 2
Pronouns should always be consistent. For example, if you are using second person
pronouns in a sentence, you should use them all the way through the sentence.
Examples
Before people start a new class, they should always read the syllabus. CORRECT
Rule 3
Examples
One of the children painted this portrait. Art is a great talent of theirs. INCORRECT
One of the children painted this portrait. Art is a great talent of his or hers. CORRECT
Each of the actresses was considered most beautiful when they were in their prime.
INCORRECT
Each of the actresses was considered most beautiful when she was in her prime.
CORRECT
Verbs
Verbs are words that show actions or states of being. For a more complete guide on
how to use verbs on the SAT and ACT, read our grammar guide.
Types of Verbs
Linking verbs are verbs that show a relationship between the subject and the predicate
nominative or predicate adjective. We can think of them as an equals sign. The most
common linking verb is the verb “to be.”
Examples
Action verbs are verbs that show an action. Many (but not all) of these verbs will take
direct objects. A verb that takes a direct object is called a transitive verb; a verb that
does not is called intransitive. Remember, you do not need to know these terms for
the test.
Examples
Conjugating Verbs
Notice that only the third person plural (he/she/it) is different. This will become very
important on the ACT and SAT because of…
Subject-Verb Agreement
When you have a singular noun or pronoun as the subject of a clause, it must be
matched by a singular verb. When you have a plural noun as the subject of a clause, it
must be matched with a plural verb.
While subject-verb agreement seems simple in theory, the SAT and ACT will try to trick
you on this by adding extra words and phrases to sentences. This can make it more
difficult to see if the verb works with the subject, as opposed to another noun in the
sentence.
Examples
My sister, despite having to carry three children, walk five miles every day.
The test will try to trick you with questions like this by putting a plural noun that is NOT
the subject next to a plural verb. To avoid falling for this, find the verb, and then ask
yourself, “who is doing this action?” In this case, who is walking five miles? It is
my sister, which is a singular noun. Therefore the sentence should read, “My sister,
despite having to carry three children, walks five miles every day.
Verb Tense
You should always try to keep tenses consistent in one sentence. This is something
the SAT and ACT will frequently test.
Example
This sentence matches past perfect with present, and therefore would be
considered incorrect.
Instead try:
This matches past perfect with past, which is ok for showing progression of time within a
sentence.
Example
This sentence mixes past tense (tasted) with present tense (are). This would be
considered incorrect.
Adjectives
What Are Adjectives?
Types of Adjectives
Examples
I like Mexican food.
He plays the French horn.
Possessive Adjectives
These adjectives are formed from the personal pronouns and are used to describe
objects that belong to a certain person.
Personal Pronoun Possessive Adjective Personal Pronoun Possessive Adjective
I/me my we our
Examples
Demonstrative Adjectives
Examples
Examples
The burned chicken did not taste very good.
Example
The basketball player is tall.
Remember that when a noun is being used as an adjective, there is no need to put a
comma in between it and the noun it is describing.
Examples
Uses of Adjectives
Adjectives are usually used to describe nouns, and are usually placed before the noun
they describe.
Example
If more than one adjective precedes a noun, they should be separated by a comma if
the order of the adjectives is not important.
Example
If you have a sentence in which the order of the adjectives IS important - meaning if you
reversed them, the sentence would not make as much sense - then do not use a
comma.
Examples
Examples
Adverbs
What Are Adverbs?
Examples
Examples
careful → carefully
easy → easily
Adjective Adverb
good well
fast fast
hard hard
late late
early early
daily daily
straight straight
Example
Examples
I daily receive a daily newspaper.
Of course, nobody would really speak this way because it’s redundant. But it shows
that daily can be an adverb describing the verb receive, and also an adjective
describing the noun newspaper.
Example
Because our casserole was smelling surprisingly badly as it baked, the food science
teacher
A
came over to ask us what we had put in it. No error.
B C D E
The answer is (A). Why? This could be confusing because “badly” is a correctly formed
adverb. However, we don’t need an adverb, we need an adjective. “was smelling” is
here used as a linking verb, and therefore we need a predicate adjective to tell us what
it smelled like. “Surprisingly” is ok as it is because it is an adverb that should be
modifying an adjective. Here is how it would look when corrected: “Because our
casserole was smelling surprisingly bad as it baked…..”
If left as is, badly would have to be describing “was smelling”, which would imply that
the casserole has a poor sense of smell.
Prepositions
What Are Prepositions?
Prepositions are words that show where someone or something is, or tells when
something is happening. They can also be used to show a few other relationships, such
as to whom you give something, or if you do something with or without something else.
Common Prepositions
Here are some of the most common prepositions in the English language:
Prepositional Phrases
Examples
Examples
If you're having problems, try taking out the other part of the compound object. This can
help make the correct form of the pronoun more obvious.
Example
Often the SAT and ACT will add unnecessary prepositional phrases to sentences to try
to make errors less obvious. Feel free to cross out prepositional phrases in order to
make sentences easier to analyze.
Anne Tyler’s novel The Accidental Tourist features a character whose obsession with
saving
A B
C
time and money are absurd, yet somehow plausible. No error.
D E
Now look with a prepositional phrase crossed out:
Anne Tyler’s novel The Accidental Tourist features a character whose obsession with
saving
A B
C
time and money are absurd, yet somehow plausible. No error.
D E
After crossing out the prepositional phrase, it becomes much more obvious that
“obsession are absurd” is incorrect.
Prepositions in Idioms
Many prepositions have to be used in a certain way with certain phrases. This is not
because one preposition is grammatically more correct, but because certain phrases in
English are idiomatically correct because they have always been said a certain way.
For example, we would say: “She fell in love with him.” We would never say, “She
fell towards love at him.”
We would say, “He is hard at work.” We would never say, “He is hard in work” with the
same meaning in mind.
Conjunctions
What are Conjunctions?
Conjunctions are words that link ideas together. There are three main types of
conjunctions: coordinating, subordinate, and correlative.
Coordinating Conjunctions
Coordinating conjunctions are probably the conjunctions that you are most familiar with.
There are seven: for, and, nor, but, or, yet, and so.
F for
A and
N nor
B but
O or
Y yet
S so
Coordinating conjunctions can be used to join similar words, phrases, or clauses.
The most commonly used conjunction is “and."
Examples
Joining words: John and Kim went to the store.
Joining phrases: The mouse ran out the door and through the garden.
Joining clauses: The mouse ran out the door, and I followed it.
Example
subject+verb +CONJ + subject+ verb
“The professor paced” and “The student sat quietly” could both be independent
sentences.
Subordinating Conjunctions
There are a lot of subordinate conjunctions, but some of the more common ones
include:
You can use the following structures to form sentences with dependent and
independent clauses:
Example
Example
Example
Correlative Conjunctions
These are very similar to coordinating conjunctions, but they must always be used in
pairs. These pairs are worth memorizing because occasionally the SAT and ACT will
test these to see if you know which words belong together.
both...and
either...or
neither…nor
whether...or
Examples
Examples